SlideShare a Scribd company logo
1 of 40
7 
Thermodynamic Relations 
7.1. General aspects. 7.2. Fundamen tals of partial differen tiation. 7.3. Some general thermodynam ic 
relations. 7.4. Entropy equation s (Tds equation s). 7.5. Equation s for internal energy and enthalp y . 
7.6. Measurable quan tities : Equation of state, co-efficien t of expansion and compressibil i ty , 
speci fic heats, Joule-Thomso n co-efficien t 7.7. Clausiu s-Clap eryo n equatio n—Hig hligh t s — 
Objectiv e Type Questio n s—Exercises. 
7.1. GENERAL ASPECTS 
In this chapter, some important thermodynamic relations are deduced ; principally those 
which are useful when tables of properties are to be compiled from limited experimental data, those 
which may be used when calculating the work and heat transfers associated with processes under-gone 
by a liquid or solid. It should be noted that the relations only apply to a substance in the solid 
phase when the stress, i.e. the pressure, is uniform in all directions ; if it is not, a single value for 
the pressure cannot be alloted to the system as a whole. 
Eight properties of a system, namely pressure (p), volume (v), temperature (T), internal 
energy (u), enthalpy (h), entropy (s), Helmholtz function (f) and Gibbs function (g) have been 
introduced in the previous chapters. h, f and g are sometimes referred to as thermodynami c 
potentials. Both f and g are useful when considering chemical reactions, and the former is of 
fundamental importance in statistical thermodynamics. The Gibbs function is also useful when 
considering processes involving a change of phase. 
Of the above eight properties only the first three, i.e., p, v and T are directly measurable. 
We shall find it convenient to introduce other combination of properties which are relatively easily 
measurable and which, together with measurements of p, v and T, enable the values of the 
remaining properties to be determined. These combinations of properties might be called ‘thermo-dynamic 
gradients’ ; they are all defined as the rate of change of one property with another while 
a third is kept constant. 
7.2. FUNDAMENTALS OF PARTIAL DIFFERENTIATION 
Let three variables are represented by x, y and z. Their functional relationship may be 
expressed in the following forms : 
f(x, y, z) = 0 
x = x(y, z) 
y = y(x, z) 
z = z(x, y) 
Let x is a function of two independent variables y and z 
x = x(y, z) 
...(i) 
...(ii) 
...(iii) 
...(iv) 
...(7.1) 
341
342 ENGINEERIN G THERMOD Y N AM ICS 
Then the differential of the dependent variable x is given by Fx I F x I 
y 
z 
z y 
...(7.2) 
where dx is called an exact differential. 
F x I F I 
H K 
If G y J 
z 
= M and H z K y 
= N 
Then dx = Mdy + Ndz ...(7.3) 
Partial differentiation of M and N with respect to z and y, respectively, gives 
M 2 x 
z yz 
M N 
or 
z y 
and 
N 2 x 
y zy 
...(7.4) 
dx is a perfect differential when eqn. (7.4) is satisfied for any function x. 
Similarly if y = y(x, z) and z = z(x, y) ...(7.5) 
then from these two relations, we have F y I F y dy = I 
x z 
dx + 
z x 
dz 
z z 
dz = x y 
dx + y 
x 
dy 
dy = G x 
I 
z 
dx + G z 
I 
F 
x 
x N 
I 
y 
F 
y 
dx  
I 
x 
dy 
Q 
F y I F y I F z I F y I F z I 
L O M P M P H K G J H G K J 
= N x 
z 
z 
x 
x 
y Q dx + z x y x 
dy 
F y I F y I F z I 
L O M P M P 
= N x 
z 
z 
x 
x 
y Q dx + dy 
y y z 
 F F 
H H G G 
I I 
K K J J 
F 
H G 
I 
K J 
or x z x = 0 
F yI F z I F y I 
or z x x y 
= – 
x 
x F z I F y I 
or y 
z 
x y z x 
= – 1 
In terms of p, v and T, the following relation holds good 
FpI FT I F v I 
v T p 
v 
T p 
= – 1 
...(7.6) 
...(7.7) 
...(7.8) 
...(7.9) 
dharm 
M-thermTh7-1.pm 5 
dx = H K G J dy  H K G J dz 
 
H G K J G H J 
K H G F 
J I 
G F I 
K H 
J 
K L O 
 H H G K G K J J H G K J 
 H H G K G K J J H G K J 
z x y 
z 
F 
H G 
I 
K J H K H K G J G J 
H K G J H K G J H K G J 
x G J 
 
 
F y F y 
H K H K J J 
z z 
H K G J H G K M J P M P 
H K H K G J G J H G K J
THERMO D Y N A M IC RELATION S 343 
7.3. SOME GENERAL THERMODYNAMIC RELATIONS 
The first law applied to a closed system undergoing a reversible process states that 
dQ = du + pdv 
According to second law, 
dQ 
ds = 
rev. 
Combining these equations, we get 
Tds = du + pdv 
or du = Tds – pdv 
The properties h, f and g may also be put in terms of T, s, p and v as follows : 
dh = du + pdv + vdp = Tds + vdp 
Helmholtz free energy function, 
df = du – Tds – sdT 
= – pdv – sdT 
Gibb’s free energy function, 
dg = dh – Tds – sdT = vdp – sdT 
Each of these equations is a result of the two laws of thermodynamics. 
Since du, dh, df and dg are the exact differentials, we can express them as F uI F uI du = 
s v 
ds + 
v s 
dv, 
h h 
dh = s p 
ds + p dp, 
F f I F f I 
df = 
v T 
dv + 
T v 
dT, 
g 
dg = G pJ 
T 
g 
dp + G T J 
p 
dT. 
...(7.10) 
...(7.11) 
...(7.12) 
...(7.13) 
Comparing these equations with (7.10) to (7.13) we may equate the corresponding co-efficients. 
For example, from the two equations for du, we have F uI F uI 
s v 
= T and 
v s 
= – p 
The complete group of such relations may be summarised as follows : F uI F hI 
s 
= T = s 
F uI F f I 
= – p = 
v s 
v T 
g 
p 
s 
= v = p 
T 
f 
H T Kv 
= – s = G T J 
p 
...(7.14) 
...(7.15) 
...(7.16) 
...(7.17) 
dharm 
M-thermTh7-1.pm 5 
T 
F 
H G 
I 
K J 
HG J K HG J 
K G 
F 
J I 
H K H 
F I 
K G J 
H H G K G K J J 
F I F I 
H H G K G K J J 
H G K J 
v H K G J 
p 
g 
F I 
F I F I 
s 
H K 
H K 
H G K J G H J 
K H G 
F 
h 
I 
J 
K G H K J 
G J 
H K
344 ENGINEERIN G THERMOD Y N AM ICS 
F T I F pI 
Also, 
HG K − HG K J J 
H K s H K 
v s 
F T I F vI 
p s p 
F p I F sI 
T v v T 
v s 
T p p 
T 
...(7.18) 
...(7.19) 
...(7.20) 
...(7.21) 
The equations (7.18) to (7.21) are known as Maxwell relations. 
It must be emphasised that eqns. (7.14) to (7.21) do not refer to a process, but simply expres s 
relations between properties which must be satisfied when any system is in a state of equilibrium. 
Each partial differential co-efficient can itself be regarded as a property of state. The state may be 
defined by a point on a three dimensional surface, the surface representing all possible states of 
stable equilibrium. 
7.4. ENTROPY EQUATIONS (Tds Equations) 
Since entropy may be expressed as a function of any other two properties, e.g. temperature 
T and specific volume v, 
s = f(T, v) F s I F s I i.e., ds = 
T v 
dT + 
v T 
dv 
s s 
or Tds = T 
T v 
dT + T 
v T 
dv ...(7.22) 
But for a reversible constant volume change 
dq = cv (dT)v = T(ds)v 
s 
or cv = T 
T 
F s I F p I 
But, 
v T 
= 
T v 
...(7.23) 
[Maxwell’s eqn. (7.20)] 
Hence, substituting in eqn. (7.22), we get 
Tds = cv 
dT + T H T Kv 
dv ...(7.24) 
This is known as the first form of entropy equation or the first Tds equation. 
Similarly, writing 
s = f(T, p) 
Tds = T H T K p 
dT + T H pKT 
dp ...(7.25) 
dharm 
M-thermTh7-1.pm 5 
s v 
F 
H G I 
K J − F 
H G 
I 
K J 
v 
F 
H 
I 
K 
G J  G J 
H  H G K G K J J 
H H G K G K J J 
HF I HF I G K G K J J 
G J 
H H G K G K J J 
F p G 
I J 
F G s J I G F s 
J 
I
THERMO D Y N A M IC RELATION S 345 
where cp = T H T K p 
s v G J 
Also G pJ 
T 
= – H T K p 
...(7.26) 
[Maxwell’s eqn. (7.21)] 
whence, substituting in eqn. (7.25) 
Tds = cpdT – T H T K p 
dp ...(7.27) 
This is known as the second form of entropy equation or the second Tds equation. 
7.5. EQUATIONS FOR INTERNAL ENERGY AND ENTHALPY 
(i) Let 
F uI 
To evaluate 
v T 
Then 
or 
But 
Hence 
u = f(T, v) F uI F uI F u du = I 
T v 
dT + 
v T 
dv = cv dT + 
v T 
dv 
let u = f (s, v) 
F uI F uI 
du = 
s 
ds + 
v 
dv 
F uI FuI F s I F uI 
= 
v 
s v v 
F uI F s I F s I F uI 
= T, 
= 
s v v 
T 
T v v s 
= – p 
F uI F p I 
= T 
v T 
T v 
– p 
...(7.28) 
...(7.29) 
This is sometimes called the energy equation. 
From equation (7.28), we get 
| | 
| | 
du = cvdT + TT F p I 
v 
− pW dv 
(ii) To evaluate dh we can follow similar steps as under 
h = f(T, p) 
dh = F h I dT + G hI 
dp 
= cpdT + H p 
I 
T 
F 
H K J 
dp 
T 
...(7.30) 
...(7.31) 
dharm 
M-thermTh7-1.pm 5 
F I F I 
H H H G K G K G K J J J 
v s H H G K G K J J 
T v T s H K H K H K H K 
H H H H G K G K G K G K J J J , J 
H H G K G K J J 
H  T K S G J V 
T p H K G J 
p 
GF s IJ 
H K 
F v I G J 
H G K J 
G J G J G J  G J 
R U 
F h G K J
346 
FhI 
To find p 
T 
; 
Then, 
But 
ENGINEERIN G THERMOD Y N AM ICS 
let h = f(s, p) 
F hI FhI 
dh = s ds + p dp 
FhI FhI F s I F hI 
p 
T 
= s p p 
T 
+ p 
s 
F hI = T, G s J = – G vJ , G hJ = v 
p T p s 
F I 
K 
Hence G pJ 
T 
= v – T H T K p 
...(7.32) 
From eqn. (7.31), we get 
dh = c 
p 
R U S V T| T W| 
dT + |v −T H 
v I 
p 
| dp ...(7.33) 
7.6. MEASURABLE QUANTITIES 
Out of eight thermodynamic properties, as earlier stated, only p, v and T are directly 
measurable. Let us now examine the information that can be obtained from measurements of 
these primary properties, and then see what other easily measurable quantities can be introduced. 
The following will be discussed : 
(i) Equation of state 
(ii) Co-efficient of expansion and compressibility 
(iii) Specific heats 
(iv) Joule-Thomson co-efficient. 
7.6.1. Equation of State 
Let us imagine a series of experiments in which the volume of a substance is measured over 
a range of temperatures while the pressure is maintained constant, this being repeated for various 
pressures. The results might be represented graphically by a three-dimens ional surface, or by a 
family of constant pressure lines on a v-T diagram. It is useful if an equation can be found to 
express the relation between p, v and T, and this can always be done over a limited range of states. 
No single equation will hold for all phases of a substance, and usually more than one equation is 
required even in one phase if the accuracy of the equation is to match that of the experimental 
results. Equations relating p, v and T are called equations of state or characteristic equations. 
Accurate equations of state are usually complicated, a typical form being 
pv = A + 
B 
 
C 
+ ...... 
where A, B, C, ...... are functions of temperature which differ for different substances. 
An equation of state of a particular substance is an empirical result, and it cannot be 
deduced from the laws of thermodynamics. Nevertheless the general form of the equation may be 
dharm 
M-thermTh7-1.pm 5 
p H K G J 
s H K G J 
F I F I F I 
H G K J 
H HG JK HG JK H G K G K J J 
H s K G J 
H p K H p K H p K 
h 
H 
F G v 
J 
I F G K J 
v v 2
THERMO D Y N A M IC RELATION S 347 
predicted from hypotheses about the microscopic structure of matter. This type of prediction has 
been developed to a high degree of precision for gases, and to a lesser extent for liquids and solids. 
The simplest postulates about the molecular structure of gases lead to the concept of the perfect 
gas which has the equation of state pv = RT. Experiments have shown that the behaviour of real 
gases at low pressure with high temperature agrees well with this equation. 
7.6.2. Co-efficient of Expansion and Compressibility 
From p-v-T measurements, we find that an equation of state is not the only useful informa-tion 
which can be obtained. When the experimental results are plotted as a series of constant 
pressure lines on a v-T diagrams, as in Fig. 7.1 (a), the slope of a constant pressure line at any 
v 
given state is T . If the gradient is divided by the volume at that state, we have a value of a 
property of the substance called its co-efficient of cubical expansion . That is, 
Fig. 7.1. Determinatio n of co-efficien t of expansion from p-v-T data. 
= 
v H T K p 
...(7.34) 
Value of can be tabulated for a range of pressures and temperatures, or plotted graphically 
as in Fig. 7.2 (b). For solids and liquids over the normal working range of pressure and tempera-ture, 
the variation of  is small and can often be neglected. In tables of physical properties  is 
usually quoted as an average value over a small range of temperature, the pressure being atmos-pheric. 
This average co-efficient may be symbolised by  and it is defined by 
v2 −v 
v (T2 −T ) 
...(7.35) 
Fig. 7.2 (a) can be replotted to show the variation of volume with pressure for various 
v 
constant values of temperature. In this case, the gradient of a curve at any state is p . When 
this gradient is divided by the volume at that state, we have a property known as the compressibility 
K of the substance. Since this gradient is always negative, i.e., the volume of a substance always 
decreases with increase of pressure when the temperature is constant, the compress ibility is 
usually made a positive quantity by defining it as 
dharm 
M-thermTh7-1.pm 5 
= 1 
F 
H G I 
K J 
p 
1 GF  v IJ 
1 1 
F 
H G 
I 
K J 
T
348 ENGINEERIN G THERMOD Y N AM ICS 
Fig. 7.2. Determinatio n of compressibi lity from p-T data. 
K = – 
F I 
K 
1 G vJ ...(7.36) 
T 
K can be regarded as a constant for many purposes for solids and liquids. In tables of 
properties it is often quoted as an average a value over a small range of pressure at atmospheric 
temperature, i.e., 
v2 −v 
v ( p2 − p ) 
When  and K are known, we have F p I F TI v 
T v v p p 
T 
v 
K = v and G vJ = – Kv, 
F p I  
T v K 
Since H 
...(7.37) 
When the equation of state is known, the co-efficient of cubical expansion and compressibility 
can be found by differentiation. For a perfect gas, for example, we have 
F 
T J = 
R 
v 
K T 
 G J 2 
and H  
 
RT 
v 
Hence = 
1 F v 
K p 
= 
pv 
= 
T 
, 
and K = – 
v HpKT 
= 
RT 
= 
p 
. 
7.6.3. Specific Heats 
Following are the three differential co-efficients which can be relatively easily determined 
experimentally. 
dharm 
M-thermTh7-1.pm 5 
K = – 1 
H K H K G J G J H G K J = – 1 
F I F I 
H G K J = 
v H p 
1 1 
F I 
G J 
T p H K p 
T 
I F I 
H G K 
p p p p 
v H T G 
I J 
R 1 
1 F v I G J 
p v 2 
1
THERMO D Y N A M IC RELATION S 349 
F uI 
Consider the first quantity 
T 
. During a process at constant volume, the first law 
informs us that an increase of internal energy is equal to heat supplied. If a calorimetric experi-ment 
is conducted with a known mass of substance at constant volume, the quantity of heat Q 
required to raise the temperature of unit mass by ΔT may be measured. We can then write : 
H ΔT K= H ΔT Kv 
v 
. The quantity obtained this way is known as the mean specific heat at constant 
volume over the temperature range ΔT. It is found to vary with the conditions of the experiment, 
i.e., with the temperature range and the specific volume of the substance. As the temperature 
range is reduced the value approaches that of H T K , and the true specific heat at constant 
volume is defined by cv = G T K . This is a property of the substance and in general its value 
varies with the state of the substance, e.g., with temperature and pressure. 
According to first law of thermodynamics the heat supplied is equal to the increase of enthalpy 
during a reversible constant pressure process. Therefore, a calorimetric experiment carried out 
with a substance at constant pressure gives us, G ΔT J = G ΔT J 
p 
which is the mean specific heat 
at constant pressure. As the range of temperature is made infinites imally small, this becomes the 
rate of change of enthalpy with temperature at a particular state defined by T and p, and this is 
h 
true specific heat at constant pressure defined by c 
p 
= 
T 
. c 
p 
also varies with the state, e.g., 
with pressure and temperature. 
The description of experimental methods of determining c and c can be found in texts on 
physics. When solids and liquids are considered, it is not easy to measure c owing to the stresses 
set up when such a substance is prevented from expanding. However, a relation between c , c , 
and K can be found as follows, from which c may be obtained if the remaining three properties 
have been measured. 
The First Law of Thermodynamics, for a reversible process states that 
dQ = du + p dv 
Since we may write u = (T, v), we have 
du = H 
du 
K v 
dT + H 
u 
K T 
dv 
 dQ = H 
uJ 
v 
dT + Tp H v J 
T W dv = cv dT + Tp H v J 
T W dv 
This is true for any reversible process, and so, for a reversible constant pressure process, 
dQ = cp(dT)p = cv(dT)p + |p H 
u 
K | (dv)p 
Hence cp – cv = 
R |S F uJ | U G F V v I 
T| p W| H J 
K T p 
Also H 
p 
Kv 
= G s J 
T 
= 
1 Sp H 
u 
KT 
V , and therefore 
cp – cv = T H T Kv H T K p 
dharm 
M-thermTh7-1.pm 5 
H G K J 
v 
F Δu I F Q I G J G J 
GF u 
IJ 
F 
I v u 
H 
J 
v 
Δh 
p 
F 
H 
I 
K 
F Q 
H 
I 
K 
F 
H G 
I 
K J 
p 
p v 
v 
p v 
v 
F I F I 
 G J G J 
T v 
F I R| F I U| R| F I U| 
T 
u u G K G K G K |S |V |S |V 
R| F I U| 
T v S G J V T W 
H v G 
I 
K 
T 
F I R| F I U| G J T 
F 
H 
I 
K T| G J v T v W| 
G F p J I G F v 
J 
I
350 ENGINEERIN G THERMOD Y N AM ICS 
Now, from eqns. (7.34) and (7.37), we have 
cp – cv = 
2Tv 
...(7.38) 
Thus at any state defined by T and v, c can be found if c ,  and K are known for the 
substance at that state. The values of T, v and K are always positive and, although may some-times 
be negative (e.g., between 0° and 4°C water contracts on heating at constant pressure), 2 is 
always positive. It follows that cp is always greater than cv. 
The other expressions for cp and cv can be obtained by using the equation (7.14) as follows : 
Since cv = H 
u 
Kv 
= H 
u 
Kv H 
s 
Kv 
We have cv = T H T J ...(7.39) 
Similarly, cp = H T K p 
= F 
h s 
s 
I 
p 
F 
T 
I 
p 
Hence, cp = T H T K p 
...(7.40) 
Alternative Expressions for Internal Energy and Enthalpy 
(i) Alternative expressions for equations (7.29) and (7.32) can be obtained as follows : G uI = T G p I – p ...(7.29) 
v 
F p I F T I F vI 
T v v p 
T 
or H T J 
v 
= – H T K p H v KT 
= + 
Kv 
= 
K 
Substituting in eqn. (7.29), we get 
H v KT 
= T 
K 
– p ...(7.41) 
F I G J 
Thus, du = cvdT + H 
T 
− pK dv ...[7.28 (a)] 
Similarly, H pJ = v – T H T K ...(7.32) 
F u I 
But by definition, T p 
= v 
Hence H pKT 
= v(1 – T) ...(7.42) 
dharm 
M-thermTh7-1.pm 5 
K 
v p 
GF IJ GF IJ GF IJ 
T s T 
GF s 
I K 
v 
F G h 
J 
I H K H K G J G J 
F G s 
J 
I F F 
H J J 
v K H T 
K T But H H G K G K J J H K G J = – 1 
GF I F I F I K 
p G J G J 
v p v  
GF IJ 
u  
K 
F G 
I 
K 
h 
T 
GF v 
IJ 
p 
G H J 
K F I G 
h 
J
THERMO D Y N A M IC RELATION S 351 
Thus 
(ii) Since 
or 
Hence 
dh = c 
p 
dT + v(1 – T) dp 
u = h – pv 
G uJ = F hI – p G vJ – v 
H p K HG p JK H p K 
T T T 
= v – vT + pKv – v 
G uJ = pKv – vT 
T 
...[7.31 (a)] 
...(7.43) 
7.6.4. Joule-Thomson Co-efficient 
Let us consider the partial differential co-efficient H p K . We know that if a fluid is flowing 
through a pipe, and the pressure is reduced by a throttling process, the enthalpies on either side of 
the restriction may be equal. 
The throttling process is illustrated in Fig. 7.3 (a). The velocity increases at the restriction, 
with a consequent decrease of enthalpy, but this increase of kinetic energy is dissipated by friction, 
as the eddies die down after restriction. The steady-flow energy equation implies that the enthalpy 
of the fluid is restored to its initial value if the flow is adiabatic and if the velocity before restriction 
is equal to that downstream of it. These conditions are very nearly satisfied in the following experi-ment 
which is usually referred to as the Joule-Thomson experiment. 
T 
Constant h 
lines 
p1,T 1 p2, T2 
   
 
Fluid 
 
 
 
 
 
p2, T2 
p1, T1 
Slope =  
 
p 
(a) (b) 
Fig. 7.3. Determination of Joule-Thomson co-efficien t. 
Through a porous plug (inserted in a pipe) a fluid is allowed to flow steadily from a high 
pressure to a low pressure. The pipe is well lagged so that any heat flow to or from the fluid is 
negligible when steady conditions have been reached. Furthermore, the velocity of the flow is kept 
low, and any difference between the kinetic energy upstream and downstream of the plug is negligible. 
A porous plug is used because the local increase of directional kinetic energy, caused by the 
restriction, is rapidly converted to random molecular energy by viscous friction in fine passages 
of the plug. Irregularities in the flow die out in a very short distance downstream of the plug, and 
dharm 
M-thermTh7-1.pm 5 
F I F I 
F I 
H p K 
F G 
I J 
T 
h
352 ENGINEERIN G THERMOD Y N AM ICS 
temperature and pressure measurements taken there will be values for the fluid in a state of 
thermodynamic equilibrium. 
By keeping the upstream pressure and temperature constant at p and T , the downstream 
pressure p is reduced in steps and the corresponding temperature T is measured. The fluid in the 
successive states defined by the values of p and T must always have the same value of the 
enthalpy, namely the value of the enthalpy corresponding to the state defined by p and T . From 
these results, points representing equilibrium states of the same enthalpy can be plotted on a T-s 
diagram, and joined up to form a curve of constant enthalpy. The curve does not represent the 
throttling process itself, which is irreversible. During the actual process, the fluid undergoes first 
a decrease and then an increase of enthalpy, and no single value of the specific enthalpy can be 
ascribed to all elements of the fluid. If the experiment is repeated with different values of p and T , 
a family of curves may be obtained (covering a range of values of enthalpy) as shown in Fig. 7.3 (b). 
The slope of a curve [Fig. 7.3 (b)] at any point in the field is a function only of the state of the 
fluid, it is the Joule-Thomson co-efficient , defined by = G p J . The change of temperature due 
to a throttling process is small and, if the fluid is a gas, it may be an increase or decrease. At any 
particular pressure there is a temperature, the temperature of inversion, above which a gas can 
never be cooled by a throttling process. 
Both c and , as it may be seen, are defined in terms of p, T and h. The third partial 
differential co-efficient based on these three properties is given as follows : FhI F p I F T I 
p T h h p 
= – 1 
F I 
h 
Hence p 
T 
= – cp ...(7.44) 
may be expressed in terms of cp, p, v and T as follows : 
The property relation for dh is dh = T ds + v dp 
From second T ds equation, we have F v I 
Tds = cp dT – T T dp 
 dh = c dT – 
L 
T F v I −v 
O 
dp 
p 
For a constant enthalpy process dh = 0. Therefore, 
| F v I | 
0 = (cp dT)h + NT T 
p W Qh 
L M R U O M |S |V 
P P 
or (cp dT)h = MST G TJ −vV dpP 
h 
  = G T J 
h 
= 
M M T G 
H J P P 
cp N 
T K p 
−v 
Q 
...(7.45) 
...(7.46) 
For an ideal gas, pv = RT ; v = 
RT 
dharm 
M-thermTh7-1.pm 5 
H G K J 
p 
NM QP 
v −T dp H K G J 
F I R| 
T 
U| 
W 
L 
N 
O 
Q 
F I L F I O 
1 1 
2 2 
2 2 
1 1 
1 1 
F T 
I 
H K 
h 
p 
H K G J H H G K G K T J J 
H K G J 
p H  T G K M J P 
p 
v 
M | H K | P 
H p K 
1  v 
p
THERMO D Y N A M IC RELATION S 353 
or 
 
F v I R v 
T 
p p T 
 = 
cp HT  
T 
−vK = 0. 
Therefore, if an ideal gas is throttled, there will not be any change in temperature. 
or 
Let h = f(p, T) 
h F h I Then dh = p dp + T dT 
F h I 
But T 
p 
F I 
K 
= cp 
h 
 dh = H pKT 
dp + cp dT 
For throttling process, dh = 0 
h p 
p 
T 
T h 
p 
1 h 
p p 
T 
h 
p 
T 
is known as the constant temperature co-efficient. 
...(7.47) 
...(7.48) 
...(7.49) 
7.7. CLAUSIUS-CLAPERYON EQUATION 
Clausius-Claperyon equation is a relationship between the saturation pressure, tempera-ture, 
the enthalpy of evaporation, and the specific volume of the two phases involved. This equa-tion 
provides a basis for calculations of properties in a two-phase region. It gives the slope of a 
curve separating the two phases in the p-T diagram. 
p 
Critical point 
Liquid 
Vapour 
Solid Triple point 
Sublimat ion 
curve 
T 
Fig. 7.4. p-T diagram. 
dharm 
M-thermTh7-1.pm 5 
H K G J = = 
F I 
H 
T H K p 
H K G J 
F I 
 0 = F I 
H K G J 
F 
H G 
I 
K J + c 
c = – 
F 
H G 
I 
K J 
F 
H G 
I 
K J 
1 v G J 
G J G J 
G J
354 ENGINEERIN G THERMOD Y N AM ICS 
The Clausius-Claperyon equation can be derived in different ways. The method given below 
involves the use of the Maxwell relation [eqn. (7.20)] 
F I F I 
H H G K G K J J 
p s 
= 
T v 
v T 
Let us consider the change of state from saturated liquid to saturated vapour of a pure 
substance which takes place at constant temperature. During the evaporation, the pressure and 
temperature are independent of volume. 
F I 
dp sg −sf 
dT vg −vf 
 H G K J = 
where, s 
= Specific entropy of saturated vapour, 
g 
s= Specific entropy of saturated liquid, 
f v= Specific volume of saturated vapour, and 
g v= Specific volume of saturated liquid. 
f Also, sg – sf = sfg = 
hfg 
and vg – vf = vfg 
where sfg = Increase in specific entropy, 
T 
vfg = Increase in specific volume, and 
hfg = Latent heat added during evaporation at saturation temperature T. 
 dp sg −sf sfg hfg 
dT vg −vf vfg T .vfg 
...(7.50) 
This is known as Clausius-Claperyon or Claperyon equation for evaporation of liquids. 
The derivative 
dT 
is the slope of vapour pressure versus temperature curve. Knowing this slope 
and the specific volume vg and vf from experimental data, we can determine the enthalpy of 
evaporation, (hg – hf) which is relatively difficult to measure accurately. 
Eqn. (7.50) is also valid for the change from a solid to liquid, and from solid to a vapour. 
At very low pressures, if we assume v − v and the equation of the vapour is taken as 
pv = RT, then eqn. (7.50) becomes 
dp hfg hfg p 
dT Tvg RT 2 
...(7.51) 
or hfg = 
RT2 dp 
p dT 
...(7.52) 
Eqn. (7.52) may be used to obtain the enthalpy of vapourisation. This equation can be 
rearranged as follows : 
dp 
= 
hfg . 
dT 
Integrating the above equation, we get 
dp 
= 
hfg dT 
h L 
T 
O 
p R NT T2 Q 
...(7.53) 
dharm 
   
= = 
p R T 2 
p z R 2 z 
dp 
~ g fg 
ln 
p 2 fg 
1 1 
 M − P 
1 1
M-thermTh7-1.pm 5
THERMO D Y N A M IC RELATION S 355 
Knowing the vapour pressure p at temperature T we can find the vapour pressure p 
1 1 2 
corresponding to temperature T2 from eqn. (7.53). 
From eqn. (7.50), we see that the slope of the vapour pressure curve is always +ve, 
since v > v and h is always +ve. Consequently, the vapour pressure of any simple compressible 
substance increases with temperature. 
— It can be shown that the slope of the sublimation curve is also +ve for any pure substance. 
— However, the slope of the melting curve could be +ve or –ve. 
— For a substance that contracts on freezing, such as water, the slope of the melting 
curve will be negative. 
+Example 7.1. For a perfect gas, show that 
c 
p 
– c 
v 
= Np H v K T Q H TK p 
 pv v H v K T 
where  is the co-efficient of cubical/volume expansion. 
Solution. The first law of thermodynamics applied to a closed system undergoing a reversible 
process states as follows : 
dQ = du + pdv 
As per second law of thermodynamics, FdQI 
ds = 
rev. 
Combining these equations (i) and (ii), we have 
Tds = du + pdv 
...(i) 
...(ii) 
Also, since 
 
Thus, 
h = u + pv 
dh = du + pdv + vdp = Tds + vdp 
Tds = du + pdv = dh – vdp 
Now, writing relation for u taking T and v as independent, we have 
du = F u I 
v 
dT + 
F 
v 
I 
T 
dv 
u 
= cv dT + 
u 
H G K J 
v T 
dv 
Similarly, writing relation for h taking T and p as independent, we have F h I FhI dh = T p 
dT + p dp 
FhI 
= cp dT + p 
T 
dp 
In the equation for Tds, substituting the value of du and dh, we have 
cv dT + F uJ 
T 
dv + pdv = cp dT + H pKT 
dp – vdp 
or c 
v 
dT + Mp FuI 
N H v K Q N H p K 
T 
P dv = c 
p 
dT – 
Lv −FhI 
T 
O 
dp 
dharm 
M-thermTh7-1.pm 5 
H T G K J 
g f fg 
L F I O F I F I M P 
u  v u G J G J G J 
H K G J 
T H G K J 
F I 
H G K J H K G J T 
H K G J 
H G 
I 
v K 
F h I 
L O 
M Q 
P
356 ENGINEERIN G THERMOD Y N AM ICS 
Since the above equation is true for any process, therefore, it will also be true for the case 
when dp = 0 and hence 
(c 
p 
– c 
v 
) (dT) 
p 
= Np HuKT Q (dv) 
p 
or (cp – cp) = Mp H v K P HTK p 
By definition, = 
1 F v I 
p 
 The above equation becomes, 
cp – cv = Mp H v K P v 
or = pv+ v 
F 
v 
I 
T 
Proved. 
u 
+Example 7.2. Find the value of co-efficient of volume expansion  and isothermal 
compressibility K for a Van der Waals’ gas obeying 
Hp  
v2 K (v −b) = RT. 
Solution. Van der Waals equation is 
Hp  
v2 K (v −b) = RT 
Rearranging this equation, we can write 
RT a 
v −b v2 
Now for we require HTK p 
. This can be found by writing the cyclic relation, 
H v K p HTKv HpKT 
= – 1 
Hence  p T 
v 
p 
= – 
H vKT 
From the Van der Waals equation, F p I R 
T 
v v −b 
Also H v KT 
= – 
(v −b)2 
+ 
v3 
Hence = 
1 F v I = 
1 M 
– 
G H 
J 
 GF  IJ 
p 
Kv P 
p 
N H v K T Q 
dharm 
M-thermTh7-1.pm 5 
L F I O 
M v P 
L F I O F I 
T 
u 
N Q 
v 
v H  T K 
L O 
T 
F u I 
N Q 
H K 
F a I 
F a I 
p = − 
F u I 
F I F I F I 
T p v 
F I F I 
H K 
H T 
K 
F  p 
I 
v 
H K = 
F I 
p RT 2 a 
L F I O 
v H T K v 
T 
p 
M M M 
P 
P P
THERMO D Y N A M IC RELATION S 357 
or = 
1 M 
− 
P 
P P 
v −b P . 
Rv 
2 
(v −b) 
. (Ans.) 
RTv 2a(v b) 
(v −b)2 v3 
L O 
Also, K = – 
1 H 
v K = – 
v M 2a 
1 
RT P = 
RTv 
2 
(v 
a v 
2 
b 
2 . (Ans.) 
v3 (v −b)2 
M M M 
M P M P 
Example 7.3. Prove that the internal energy of an ideal gas is a function of temperature alone. 
Solution. The equation of state for an ideal gas is given by 
p = 
RT 
But H v KT 
= T HT Kv 
− p [Eqn. (7.29)] 
= T 
v 
− p = p – p = 0. 
Thus, if the temperature remains constant, there is no change in internal energy with 
volume (and therefore also with pressure). Hence internal energy (u) is a function of temperature 
(T) alone. ...Proved. 
Example 7.4. Prove that specific heat at constant volume (c ) of a Van der Waals’ gas is a 
function of temperature alone. 
Solution. The Van der Waals equation of state is given by, 
RT a 
v −b v2 
or H p Kv 
= v −b 
or G 2p J = 0 
v 
Now 
Fdc I 
= T H 
p 
K 
 
T 
Hence 
T H K G J  G J 
F 
v 
I 
H K v G J 
T 
= 0 
v 
Thus c of a Van der Waals gas is independent of volume (and therefore of pressure also). 
Hence it is a function of temperature alone. 
+Example 7.5. Determine the following when a gas obeys Van der Waals’ equation, 
Hp  
v2 K (v – b) = RT 
(i) Change in internal energy ; (ii) Change in enthalpy ; 
(iii) Change in entropy. 
Solution. (i) Change in internal energy : 
The change in internal energy is given by 
du = cvdT + MT HTKv 
− pP dv 
dharm 
M-thermTh7-1.pm 5 
v 
R 
− RT 2a 
 
L 
N 
O 
Q 
3 2 − − 
F  I 1 M P v − b 
v p T − N Q 
3 
) 
− 2 ( − ) 
v 
F u I F  p I 
R 
v 
p = − 
F I 
T 
R 
F I 
H T 2 K 
F I 
dv 
2 
2 
c v 
v 
F a I 
L F  p I O 
N Q
358 
But, 
 
 
 
 
 
 
 
 
 
 
 
 
 
ENGINEERIN G THERMOD Y N AM ICS 
F p I L  R RT a UO 
T 
v 
= = R 
T v −b v2 
v −b 
z 2 
z 2 
z 2 L 
F G R I 
J du c 
v dT  
N T 
H − K Q 
M M P − p 
P 
NM T W QP 
O 
dv 
= cv z2 
dT z2 L 
1 1 1 
T 
F 
v −b 
I 
− 
R 
v −b 
− 
O 
dv 
a V 
= cv 
2 
dT  
2 M U MP 1 
v −b 
2 R RT 
H K v G J S 
RT RT 
− 
v −b 
 
a P dv 
2 1 1 
z 2 z 
2 
L O 
N Q c dT .dv 
1 1 
u 
2 
– u 
1 
= c 
v 
(T 
2 
– T 
1 
) + a 
F 1 
− 
1 I 
. (Ans.) 
1 2 
(ii) Change in enthalpy : 
The change in enthalpy is given by 
dh = c dT + Mv −T F v I P dp 
p 
FhI 
= 0 + v – T 
F v I 
...(1) 
T p 
Let us consider p = f(v, T) 
 dp = H v KT 
dv + HT Kv 
dT 
 (dp)T = H v KT 
dv + 0 as dT = 0 ...(2) 
From equation (1), 
(dh)T = Nv −T HT K pQ (dp)T. 
O 
H 
Substituting the value (dh) of (dp) = v from −T Heqn. K (2), we Kget T NT v p 
T 
dv 
F I F I 
M p v  p P M P 
= Nv H vKT 
−T HT K p H v KT Q dv ...(3) 
Using the cyclic relation for p, v, T which is F v I FTI FpI 
T 
p 
p 
v 
v 
T 
 
F v I G pJ −G p J 
p T v 
dharm 
M-thermTh7-1.pm 5 
H K 
TS − WV N Q M P 
v 
v b 
v 
a 
v 
z  z 
v v G J 
= v 2 
H K 
L O 
p NM H T K QP 
H p K H  T K 
F p I F p I 
F p I 
L F v I O M P M P 
T 
L M M v P p P Q 
 
L F I F I F I O 
H K H H G K G K J J − 1 
F I F I H T K H v K H T K
THERMO D Y N A M IC RELATION S 359 
Substituting this value in eqn. (3), we get 
(dh) = Mv FpI T F p I P dv ...(4) 
T v 
For Van der Waals equation 
p  LF RT I a O 
T v v −b v2 
T 
= – 
(v −b)2  
v3 ...(5) 
p R 
HT Kv 
= 
M P 
T MH 
RT 
− 
a 
K Pv 
= 
v −b 
...(6) 
R 
− 
Substituting the values of eqns. (5) and (6) in equation (dh) (1), we get T = Mv 
(v −b)2 
 
v 
aU 
T H v 
R 
bK P dv 
 
2 2 v 
1 
(dh)T = – RT 
1 (v −b)2 dv + 2a 
F I G J 
2 2 dv 
1 
dv 
+ RT 
1 (v −b) 
(h2 – h1)T = – RT Mloge 
F 
v −bJ −b R 
v 2 
G − I 
H K TS 1 − 
1 
v −b 
v −b 
WV 
UP 
– 2a 
F 1 
− 
1 I 
RT log 
F v2 −b I 
2 1 1 
= bRT M(v2 −b) 
− 
(v1 −b) Q – 2a M v2 
− 
v1 
P . (Ans.) 
(iii) Change in entropy : 
The change in entropy is given by 
ds = cp 
dT 
 
H 
p Kv 
. dv 
For Van der Waals equation, 
p R 
T 
v 
v −b 
...as per eqn. (6) 
 ds = cv T 
 
v −b 
dv 
2 
 ds = cv 
2 
MdT P R 
2 
(v 
dv 
b) 
 s2 – s1 = cv loge M 
1 
P + R loge N 
v2 −b 
Q . (Ans.) 
Example 7.6. The equation of state in the given range of pressure and temperature is 
given by 
v = 
RT 
− 
C 
where C is constant. 
Derive an expression for change of enthalpy and entropy for this substance during an 
isothermal process. 
dharm 
M-thermTh7-1.pm 5 
F I 
F I 
L O 
T N H  v K H  T K Q 
F I H v K H K 
= − N Q M RT 2a P 
F I  L F I O 
N G J v − b 2 
v Q 
S RT T 
V W − 
L 
NM 
O 
QP 3 
2 
z z z z 
v2 b 
1 2 1 
L 
N 
O 
Q 
v v v b e H K H − K G J G J 
L 1 1 O L O 
N P 
1 1 
N Q 
T T 
H K  
dT R 
z 
z L O z 
1 
1 N T Q 1 − 
L T O L O 
T 
2 
N Q v 1 −b M P 
p T 3
360 ENGINEERIN G THERMOD Y N AM ICS 
Solution. The general equation for finding dh is given by 
dh = cp dT + 
L 
v −T F 
M T 
P M P 
I 
p 
O 
dp 
2 
dh = 0 + z2 MR 
| 
T| 
v −T F 
T 
I 
p 
UP 
T 
as dT = 0 for isothermal change. 
From the given equation of state, we have 
HTK = 
R 
 
3C 
...(i) 
 h2 – h1 = z2 
L M 
O 
N Q 
RF RT 
− 
C I − 
| 
RT 
− 
3C U 
dpPT 
= M 
2 
1 
| 
T| V 
G − 
4C J dpP 
T 
 − 
4C 
[( p2 − p )] 
The general equation for finding ds is given by 
ds = cp 
dT 
−H v K p 
dp 
z2 
ds = 
L 
− 
2F 
M M T 
Q 
I 
p 
dp 
O 
T 
as dT = 0 for isothermal change. 
(s2 – s1) = M 2 
Substituting the value from eqn. (i), we get 
−G R 
 
3C I 
dp 
O 
H p1 K G J 
H T 4 K 
4 1 
= – R loge 
Fp2 I 
– 
F3CI 
(p2 – p1) (Ans.) 
Example 7.7. For a perfect gas obeying pv = RT, show that c and c are independent of 
pressure. 
Solution. Let s = f(T, v) 
Then ds = HT Kv 
dT + Hv KT 
dv 
Also u = f(T, v) 
Then du = HT Kv 
dT + H v KT 
dv = cv dT + H vKT 
dv 
Also, du = Tds – pdv 
Tds – pdv = cv dT + HuKT 
dv 
ds = cv T 
 
T MF 
v 
I 
T 
 pP dv 
dharm 
M-thermTh7-1.pm 5 
v 
N H K Q 
z 
L O 
1 
v 
H G K S J 
V 
N W| Q 1 M P 
F v I 
p p T 4 
H p T p T G K S J 
| 
W| 
M P 3 3 1 
N L z 
H F K T 3 I Q 
O 1 
T 
3 T 
F I 
T T 
1 H G K J 
N 
z 
v 
P 1 
P L F z H K J N Q 
P 
p T T 
v p 
F s I F s I 
F u I F u I F u I 
F I 
v 
dT L u O 
N H  K Q 
1
THERMO D Y N A M IC RELATION S 361 
Equating the co-efficients of dT in the two equations of ds, we have 
F I  H G K J 
cv s 
T T v 
c 
v 
= T H s K 
Fcv I 2 s 
v T Tv 
From eqn. (7.20), F s I F p I 
v 
T 
T 
v 
2 s 2 p 
vT T2 
Fcv I F 2p I 
v T T2 
v 
 H G K J 
Also p = 
RT 
...(Given) 
F p I R 
T v 
F 
p I 
T 
v 
F I 
H K 
0 or G  v J 
T 
0 
2 
c 
v 
This shows that cv is a function of T alone, or cv is independent of pressure. 
Also, cp = T H s K p 
H p K T 
Tp 
From eqn. (7.21), 
F sI −F v I 
T p 
2s 2v 
pT T2 
p 
G cp J −T G 2 
2 J 
T p 
Again, v = 
R 
F v I R 
T p F 2v I 
and T2 
p 
= 0 ; 
...(Given) 
H p KT 
= 0 
This shows that cp is a function of T alone or cp is independent of pressure. 
dharm 
M-thermTh7-2.pm 5 
p 
H K G J  
H G K J 
F I 
T 
v 
H K G J  T 
H K  H K 
F I 
v 
H K G J H G K J T 
v 
H K 
 
v 
 
H G K J 
2  
F I 
T 
F  I G J 
c p  s 
T 
2 
H p K H T K 
F I − H K G J 
F I F I 
v 
H p K H  T 
K 
p 
F G c I p 
J
362 ENGINEERIN G THERMOD Y N AM ICS 
Example 7.8. Using the first Maxwell equation, derive the remaining three. 
Solution. The first Maxwell relation is as follows : 
F T I F  p I 
H v Ks 
 −H s Kv 
(1) Using the cyclic relation 
T v s 
H v Ks 
. H s KT 
. HT Kv 
= – 1 
s T s 
 Hv KT 
= – H v Ks 
. HT Kv 
Substituting the value from eqn. (i) in eqn. (ii), we get 
s p s 
Hv KT 
= H s Kv 
. HT Kv 
Using the chain rule, 
p s T 
H s Kv 
. HT Kv 
. H p Kv 
= 1 
Substituting the value of eqn. (iv) in eqn. (iii), we get 
F s I F p I 
Hv KT 
= HT Kv 
This is Maxwell Third relation. 
(2) Again using the cyclic relation 
vI 
F H s I F I F K H p 
K H K 
p 
v 
. . = – 1 
v 
s 
s 
p 
v FpI F v I 
s 
 H K = – H K H K . 
p s 
v 
p 
s 
Substituting the value from eqn. (i) into eqn. (v) 
v T v 
HsK p 
= H v Ks 
. HpKs 
Again using the chain rule, 
T v p 
H v Ks 
. HpKs 
. HT Ks 
= 1 
Substituting the value of (vi) into (v), we get 
F v I F  T I 
HsK p 
H p Ks 
This is Maxwell second relation. 
(3) HTK p 
. H p Kv 
. H vKT 
= – 1 
...(i) (Eqn. 7.18) 
...(ii) 
...(iii) 
...(iv) 
...(v) 
...(vi ) 
dharm 
F I F I F I 
F I F I F I 
F I F I F I 
F I F I F I 
F I 
F I F I F I 
F I F I F I 
F I F I F I 
v T p
M-thermTh7-2.pm 5
THERMO D Y N A M IC RELATION S 363 
F v I F p I F v I 
T 
H K = – H K H K . 
p T 
v 
p 
T 
F p I F s I F s I F v I 
= – H s Kv HT Kv HpKT H s KT 
Substituting the value from eqn. (i), we get F v I F TI F s I F s I F vI 
p v s T v p 
H T K = G H K J G H K J G H J K T 
H G K J 
s T 
RS F T I F v I F s I UV F s I F s 
I 
= TH v Ks 
. H sKT 
. HTKvW H pKT 
= – HpKT 
F I 
 H v K p 
= – HpKT 
This is Maxwell fourth relation. 
Example 7.9. Derive the following relations : 
(i) u = a – T F 
a 
T 
I (ii) h = g – T F g I 
p 
(iii) cv = – T G 2a J (iv) cp = – T 
F 2 g J 
G K 2 T 
T H  
v p 
H  K 2 
where a = Helmholtz function (per unit mass), and 
g = Gibbs function (per unit mass). 
Solution. (i) Let a = f(v, T) 
Then da = H v KT 
dv + HT Kv 
dT 
Also da = – pdv – sdT 
Comparing the co-efficients of dT, we get 
HT Kv 
= – s 
Also 
or 
Hence 
(ii) Let 
Then 
Also 
a = u – Ts 
u = a + Ts = a – T HT Kv 
u = a – T HT Kv 
. (Ans.) 
g = f(p, T) 
dg = HpKT 
dp + HTK p 
dT 
dg = vdp – sdT 
Comparing the co-efficients of dT, we get 
HTK p 
= – s 
dharm 
M-thermTh7-2.pm 5 
F I 
F I 
F I F I 
 
T 
F s I 
H K 
v H  T K 
F I I 
F a I F a I 
F a I 
a 
a 
g g 
F g I
364 ENGINEERIN G THERMOD Y N AM ICS 
Also h = g + Ts = g – T HTK p 
Hence h = g – T HTK p 
. (Ans.) 
(iii) From eqn. (7.23), we have 
c 
v 
= T H s K 
Also 
F a I 
= – s 
v 
or HT Kv 
2a 
= – HT2 Kv 
From eqns. (i) and (ii), we get 
c 
v 
2a 
= – T G T2 J 
v 
. (Ans.) 
(iv) From eqn. (7.26), we have 
cp = T H s K p 
Also HTK = – s 
or 
F 
T 
I 
p 
s G J T 2 
= – H 
2g 
K p 
From eqns. (i) and (ii), we get 
2 g 
cp = – T HT2 K p 
. (Ans.) 
...(i) 
...(ii) 
...(i) 
...(ii) 
Example 7.10. Find the expression for ds in terms of dT and dp. 
Solution. Let s = f(T, p) 
Then ds = H s K p 
. dT + HpKT 
dp 
As per Maxwell relation (7.21) 
HpKT 
= – HTK p 
Substituting this in the above equation, we get 
ds = H s K p 
dT – H v K p 
. dp ...(i) 
The enthalpy is given by 
dh = cpdT = Tds + vdp 
Dividing by dT at constant pressure 
HTK p 
= cp = T HTK p 
+ 0 (as dp = 0 when pressure is constant) 
dharm 
M-thermTh7-2.pm 5 
F I 
H T K 
F I F I 
F I 
F I 
F I 
p 
F I 
F I 
F g I 
F g I 
T 
v 
s G J 
H K 
T 
g 
H K 
G J 
F I F s 
I 
 T 
F s I F v I 
F I F I 
T  T 
F h I F s I
THERMO D Y N A M IC RELATION S 365 
Now substituting this in eqn. (i), we get 
But 
ds = cp 
dT 
F I 
−H 
s 
K . dp ...(ii) 
1F v I 
v T 
p 
Substituting this in eqn. (ii), we get 
ds = cp 
dT 
– vdp (Ans.) 
T 
Example 7.11. Derive the following relations : 
F T I F T 
I 
(i) H p K = 
Tv 
(ii) H v K = – 
T 
c K 
. 
where = Co-efficient of cubical expansion, and 
K = Isothermal compressibility. 
Solution. (i) Using the Maxwell relation F (7.19), we have T I FvI F v I FTI 
s s 
p T 
p 
s 
p 
Also cp = T HTK 
From eqn. (7.34), = 
v 
F 
T 
I 
p 
H p Ks 
 
vT 
i.e., H p Ks 
 
c 
 . (Ans.) 
(ii) Using the Maxwell relation (7.18) FT I FpI F p I FT I 
s s 
v 
T 
v 
s 
v 
Also cv = T HT K 
K = – 1 F v I 
FTI T F p I 
v c T 
Also FpI F v I FTI = – 1 
T p v 
i.e., H p Kv 
= – HvKT HTK p 
= – H−vK K v = 
K 
(Eqn. 7.23) 
(Eqn. 7.36) 
dharm 
M-thermTh7-2.pm 5 
p 
G J 
T T 
= H K 
H p K = H  K = H K H K 
F I 
F I 
F I 
F I 
v H  p K 
s v v H K H K 
F I F I F I F I 
c p 
s s v 
s 
p 
1 v 
H K 
 T 
c p 
 T Tv 
p 
H v K = – H s K = – H K H K 
v 
T 
Then = – 
H v K H T K H p K 
T 
p v 1 
366 ENGINEERIN G THERMOD Y N AM ICS 
F I 
v c K v 
 HT Ks 
= 
−T 
. (Ans.) 
+Example 7.12. Derive the third Tds equation 
F T I 
Tds = cv H p Kv 
dp + cp 
F 
v 
I 
p 
dv 
and also show that this may be written as : 
Solution. Let 
Then 
or 
But 
Hence 
Also 
and 
Tds = 
cv 
Kdp + 
c 
v 
dv. 
dss == f(p, v) F 
p 
T 
H G K J 
I dp + HsK dv 
s 
Tds = T F s I dp + T H 
s 
K dv 
v 
G J 
p F I F I 
T p T v 
= T H s Kv HTKv 
F I 
+ T F s I 
p 
FTI 
p 
dv 
H s Kv 
= T and H s K p 
= T 
Tds = cvHpK dp cp HvK dv ...Proved. 
F 
I 
F I F I 
p 
= FpI 
− 
F v I = – H v K 
F 
p 
I 
= 
K 
p H K H K 
v 
T 
1 T 
T 
p 
H v K p 
= v 
Substituting these values in the above Tds equation, we get 
Tds = 
c 
 
K 
dp  
v 
dv ...Proved. 
Example 7.13. Using Maxwell relation derive the following Tds equation 
Tds = cp dT – T HTK p 
dp. (U.P.S.C. 1988) 
Solution. 
where cp = T H TK p 
Also, 
s = f (T, p) 
Tds = T HTK p 
dT + T HpKT 
dp 
H s KT 
= – F v I 
p 
...(i) 
......Maxwell relation 
Substituting these in eqn. (i), we get 
Tds = cp dT – T HTK p 
dp. (Ans.) 
dharm 
 
p 
H K v 
F I 
p 
H K v  
v p 
H K H K 
F I 
v p 
F I 
F I 
F I F I 
F I 
T 
c v 
 T 
c p 
T T 
T 
v H K H K 
v 
T  
T 1 
c v p 
F v I 
GF s 
IJ 
s s 
F I 
p H T K 
F v I
M-thermTh7-2.pm 5
THERMO D Y N A M IC RELATION S 367 
Example 7.14. Derive the following relations 
F I 
FT I TG p J − p 
H K u 
v 
Solution. 
FT I 
H  K 
T 
c 
v 
= . 
v 
can be expressed as follows : 
u 
H v K = F vI 
F I F I 
Kv = – F uI 
T u 
u v T 
− H H K 
H K H K 
u 
T 
T 
v 
H v K 
F T I 
u 
Also Tds = du + pdv 
or du = Tds – pdv 
or HuK = T H 
s 
K – p H v K 
or 
v 
F uI 
T v T 
= T 
F s I 
– p 
T T 
or H u Kv 
= T H 
s 
Kv 
F I 
T T 
Dividing eqn. (i) by eqn. (ii), we get 
 
F I K 
F I 
FT I THvK − p 
v 
H K = 
s 
T 
u s 
T 
v 
s 
Also cv = T HT K 
and F 
s 
H G K J 
v 
I 
T 
= F 
p 
H K 
F 
H 
T 
I 
v 
u 
T 
... Maxwell relation 
Substituting these value in eqn. (iii), we get 
TG p J − p 
v 
u 
= 
cv 
v ...Proved. 
+Example 7.15. Prove that for any fluid 
(i) HhKT 
= v HpKT 
+ T H p Kv 
(ii) HhKT 
= v – T HTK p 
Show that for a fluid obeying van der Waal’s equation 
p = 
v −b 
– 
v2 
...(i) 
...(ii) 
...(iii) 
where R, a and b are constants 
h (enthalpy) = 
RTb 
− 
2a 
+ f(T) 
where f(T) is arbitrary. 
dharm 
F I F I F I 
H v K H v K 
I I 
T 
H 
v 
F I H K 
T  T 
I 
K 
F I F I F I F I F I 
v v T p 
v 
RT a 
v −b v
M-thermTh7-2.pm 5
368 ENGINEERIN G THERMOD Y N AM ICS 
Solution. We know that 
ds = 
c v  p 
T 
dT  HT K dv 
Also dh = Tds + vdp = T M v dT F p I dvP + vdp 
v 
i.e., dh = c 
v 
dT + T H p Kv 
+ dv + vdp 
Putting dT = 0, we get 
G hJ T G p J v G pJ ...Proved. 
T v T 
(ii) HhKT 
hKT H v KT 
H  
= NT F 
  
 
p I 
v 
vF pI 
vJ 
T Q G  
i.e., 
h GF IJ G J T p 
F 
p 
I 
T 
= T H 
p 
K 
F vI 
+ v 
Also F p I 
v H 
vJ 
T 
= – H v K p 
 Eqn. (i) becomes 
HpKT 
= v – T H vKp 
...Proved. 
[Eqn. (7.24)] 
...(i) 
Now 
and 
 
 
 
 
 
 
 
 
i.e., 
RT a 
v −b v2 
p RT 2a 
(v −b)2 v3 
F p I R 
T v −b 
FhI L −RT 2aO F R I 
T (v −b)2 v3 v −b 
RTv 2a RT −RTv RT 2a 
2 
(v −b ) 
v 
2 v −b (v −b)2 v −b v2 
−RTv RT(v −b) 2a −RTv RTv −RTb 2a 
(v −b)2 v2 (v −b)2 v2 
FhI −RTb 2a 
v 
T (v −b)2 v2 
or 
RTb 2a 
v −b v 
This shows h depends on T and v. 
...Proved. 
Example 7.16. Derive the following relations : 
(i) FhI = v – T 
v 
FTI FuI H p = – c(ii) = T K – p T p 
h T 
dharm 
M-thermTh7-2.pm 5 
v 
F I 
L 
N 
O 
Q 
F I 
F I F I F I 
F I F I F I L O F I 
H K v T 
F I F I 
F I F I 
p = − 
F I H v K = –  
H K v = 
N Q H K 
=  =  
H K =  
h = −  f(T) 
c 
T H  T K 
T 
H  v K H T K H  v K 
 
p v p 
M G H J K G H J K P 
T v H p K 
T H K 
H K G T J G p K  T 
 h 
 T 
T 
H v K = v M  P T 
= –   =   
F I F I 
H T K 
H K p H K H K p p v T 
v
THERMO D Y N A M IC RELATION S 369 
With the aid of eqn. (ii) show that 
F u I F v I F I 
H pKT 
= – T HTK – p HpKT 
The quantity cp 
p 
FTI is known as Joule-Thomson cooling effect. Show that this cooling 
h 
effect for a gas obeying the equation of state (v – b) = 
RT 
– 
C 
is equal to H 
3C 
K −b. 
Solution. We know that F 
p 
I 
= – cp 
Also = 
cp 
L 
HT K p 
−v 
O 
 
FhI 
= – T F v I −v 
O 
= v – T F v I 
T p p 
Also = H p Kh 
 H pKT 
= – cp H p Kh 
. 
...[Eqn. (7.44)] 
...[Eqn. (7.46)] 
... Proved. 
(ii) Let 
Also 
u = f(T, v) 
du = HT Kv 
dT + H v KT 
dv 
= c 
v 
dT + H v KT 
dv ...(i) 
du = Tds – pdv 
Substituting the value of Tds [from eqn. 7.24], we get 
du = c 
v 
dT + T 
F p I 
G J v 
N  T Q 
dv – pdv 
= cv dT + MT H p Kv 
− pP dv ...(ii) 
From (i) and (ii), we get 
HuK = T HT K – p ...Proved. 
FuI FuI F v I 
p 
T 
v 
T 
p 
T 
or 
F 
p 
I 
u v = HpKp 
T 
T 
GF IJ 
T HTKv 
− pQ 
or H pKT 
= T F 
H G J K G H J F I T 
K H K 
I 
v 
F 
p 
I 
T 
– p G pJ 
T 
...Proved. 
dharm 
M-thermTh7-2.pm 5 
H K T 
NM QP 
L 
NM QP 
F I 
F I F I 
F I F I 
F I 
v 
H p K 
F I 
p T 2 
T G 2 J 
h 
T 1 M F  v I P 
H p K H  T K M P H T K 
T 
h T 
u u 
u 
H K G J 
T 
L F I O 
F I F I 
v 
T 
G p 
J 
v 
Also H K = H K H K G J G J 
F I L F I O 
H K G J 
G J G  J N 
M P 
u  p  v 
v
370 
We know that 
or 
Also 
and 
FpI F v I FTI 
v 
H K H K H K = – 1 
T 
T 
p 
p 
v 
I 
K J − F F 
p v v 
T p T 
F 
p 
I 
H K H K G J G J u v 
T 
= – T F 
v 
T 
I 
p 
– p F 
p 
I 
T 
= 1 
L O M P N Q 
c T p H  K 
M 
F uI 
p 
−vP 
ENGINEERIN G THERMOD Y N AM ICS 
...Already proved. 
...[Eqn. (7.46)] 
Now v – b = RT 
− 
C ...[Given] 
HTK p 
= 
R 
 
2C 
Substituting this value in the expression of above, we get 
= 
1 L 
T F H 
G I J N 
R 
 
2C 
K −v 
O 
M Q 
or cp = T G R 
H  
p 3 K T p T 2 
2CJ – 
RT 
 
C 
– b = 
3 
2 
−b 
or cp H p Kh 
= 
T2 
– b ...Proved. 
Example 7.17. The pressure on the block of copper of 1 kg is increased from 20 bar to 800 
bar in a reversible process maintaining the temperature constant at 15°C. Determine the following : 
(i) Work done on the copper during the process, 
(ii) Change in entropy, (iii) The heat transfer, 
(iv) Change in internal energy, and (v) (cp – cv) for this change of state. 
Given :  (Volume expansitivity = 5 × 10–5/K, K (thermal compressibility) = 8.6 × 10 12 m2/N 
and v (specific volume) = 0.114 × 10–3 m3/kg. 
Solution. (i) Work done on the copper, W : 
Work done during isothermal compression is given by 
2 
W = pdv 
1 
The isothermal compressibility is given by 
K = – 
1 F v I 
T 
 dv = – K(v.dp)T 
2 2 
 W = – pKv.dp = – vK pdp 
1 1 
Since v and K remain essentially constant 
 W = – vK (p2 
2 – p1 
2) 
= – 
0.114 10−3 8.6 10−12 
[(800 × 105)2 – (20 × 105)2] 
dharm 
M-thermTh7-2.pm 5 
F 
H G 
H H G G I I 
K K J J 
v T p 
H K 
T 
p T 2 
F v I 
p T 3 
c p 3 p T 
P 
F I 
C 
T 
GF IJ T 3 C 
– 
z 
v H  p K 
z z 
2 
2
THERMO D Y N A M IC RELATION S 371 
= – 
0.114 8.610−15 
× 1010 [(800)2 – (20)2] 
= – 
0.114 8.6 10−5 
(640000 – 400) = – 3.135 J/kg. (Ans.) 
The negative sign indicates that the work is done on the copper block. 
(ii) Change in entropy : 
The change in entropy can be found by using the following Maxwell relation : 
G pJ 
T 
= – H v K p 
F I = – 
F I 
v 
F 
T 
I 
p 
= – vas 
1 H v K p 
=   
 (ds) 
T 
= – v (dp) 
T 
Integrating the above equation, assuming v and  remaining constant, we get 
s2 – s1 = – v (p2 – p1)T 
= – 0.114 × 10–3 × 5 × 10–5 [800 × 105 – 20 × 105] 
= – 0.114 × 10–3 × 5 (800 – 20) = – 0.446 J/kg K. (Ans.) 
(iii) The heat transfer, Q : 
For a reversible isothermal process, the heat transfer is given by : 
Q = T(s2 – s1) = (15 + 273)(– 0.4446) = – 128 J/kg. (Ans.) 
(iv) Change in internal energy, du : 
The change in internal energy is given by : 
du = Q – W 
= – 128 – (– 3.135) = – 124.8 J/kg. (Ans.) 
(v) cp – cv : 
The difference between the specific heat is given by : 
c 
p 
– c 
v 
= 
2Tv 
... [Eqn. (7.38)] 
(5 10−5)2 (15 273)  0.114 10− 3 
= 
8.6 10−12 = 9.54 J/kg K. (Ans.) 
Example 7.18. Using Clausius-Claperyon’s equation, estimate the enthalpy of vapourisation. 
The following data is given : 
At 200°C : vg = 0.1274 m3/kg ; vf = 0.001157 m3/kg ; H dT K = 32 kPa/K. 
Solution. Using the equation 
F dp I fg 
dT T (vg −v ) 
where, hfg = Enthalpy of vapourisation. 
Substituting the various values, we get 
32 × 103 = 
(200 273)(0.1274 −0.001157) 
 hfg = 32 × 103 (200 + 273)(0.1274 – 0.001157) J 
= 1910.8 × 103 J/kg = 1910.8 kJ/kg. (Ans.) 
dharm 
M-thermTh7-2.pm 5 
2 
2 
F I 
H K 
s v v 
T H K v T 
K 
F dp I G J 
H G K J = 
h 
s f 
h fg
372 ENGINEERIN G THERMOD Y N AM ICS 
Example 7.19. An ice skate is able to glide over the ice because the skate blade exerts 
sufficient pressure on the ice that a thin layer of ice is melted. The skate blade then glides over 
this thin melted water layer. Determine the pressure an ice skate blade must exert to allow 
smooth ice skate at – 10°C. 
The following data is given for the range of temperatures and pressures involved : 
hfg( ice) = 334 kJ/kg ; vliq. = 1 × 10 m3/kg ; vice = 1.01 × 103 m3/kg. 
Solution. Since it is a problem of phase change from solid to liquid, therefore, we can use 
Clausius-Claperyon equation given below : 
dp fg 1 
dT fg T 
Multiplying both the sides by dT and integrating, we get 
p2 
dp fg 
T2 dT 
1 v g 1 
T 
or (p2 – p1) = 
fg 
loge GT2 J ...(i) 
But at p1 = 1 atm., t1 = 0°C 
Thus, p1 = 1.013 bar, T1 = 0 + 273 = 273 K 
p2 = ?, T2 = – 10 + 273 = 263 K 
Substituting these values in eqn. (i), we get 
334 103 
(p2 – 1.013 × 10 ) = 
1− 1.01 
× loge 273 
= 334 
 
0 
H 263 K 
. 103 
× loge 
F273I 
= 12.46 × 105 N/m2 
or p2 = 12.46 × 105 + 1.013 × 105 
= 13.47 × 105 N/m2 or 13.47 bar. (Ans.) 
This pressure is considerably high. It can be achieved with ice skate blade by having only a 
small portion of the blade surface in contact with the ice at any given time. If the temperature 
drops lower than – 10°C, say – 15°C, then it is not possible to generate sufficient pressure to melt 
the ice and conventional ice skating will not be possible. 
Example 7.20. For mercury, the following relation exists between saturation pressure 
(bar) and saturation temperature (K) : 
log10 p = 7.0323 – 3276.6/T– 0.652 log10 T 
Calculate the specific volume v 
g 
of saturation mercury vapour at 0.1 bar. 
Given that the latent heat of vapourisation at 0.1 bar is 294.54 kJ/kg. 
Neglect the specific volume of saturated mercury liquid. 
Solution. Latent heat of vapourisation, hfg = 294.54 kJ/kg (at 0.1 bar) 
Using Clausius-Claperyon equation 
dp h fg h 
fg 
dT v gT (vg −v )T 
Since vf is neglected, therefore eqn. (i) becomes 
dp fg 
dT vgT 
...(given) 
...(i) 
Now, log 
10 
p = 7.0323 – 
3276.6 
– 0.652 log 
10 
T 
dharm 
M-thermTh7-2.pm 5 
= 
h 
v . 
z h 
= z 
p f T 
h F I 
fg 
v 
H T1 K 
F I 5 b g 
263 H K 
0 1 
= = 
f f 
= 
h 
T
THERMO D Y N A M IC RELATION S 373 
Differentiating both sides, we get 
1 dp 3276.6 0.652 
2.302p dT T 2.302 T 
or 
dT 
= 2.302 × 3276.6 × 
T2 – 0.652 
T 
...(ii) 
From (i) and (ii), we have 
or 
fg 
vgT 
= 2.302 × 3276.6 × 
T2 – 0.652 
T 
...(iii) 
We know that 
At p = 0.1 bar, 
log 
10 
p = 7.0323 – 
3276.6 
– 0.652 log 
10 
T ... (given) 
log10 (0.1) = 7.0332 – 
3276.6 
– 0.652 log10 T 
– 1 = 7.0323 – 
3276.6 
– 0.652 log10 T 
or 0.652 log10 T = 8.0323 – 
3276.6 
or log10 T = 12.319 – 
5025.4 
Solving by hit and trial method, we get 
T = 523 K 
Substituting this value in eqn. (iii), we get 
294.54 103 
vg 523 
= 2.302 × 3276.6 × 
0.1105 
– 0.652 × 
0.1105 
(523) 
563.17 
= 275.75 – 12.46 
g 
i.e., vg = 2.139 m3/kg. (Ans.) 
HIGHLIGHTS 
1. Maxwell relations are given by 
FTI 
= – FpJ ; FTJ = 
F 
s 
I 
v s H  H  G I G I 
F p I FsI F v I F sI 
T 
v 
v 
T 
T 
p 
p 
T 
2. The specific heat relation s are 
cp – cv = 
vT2 
; cv = T 
F s I 
v 
; cp = T G T J 
p 
. 
3. Joule-T homson co-efficien t is expressed as 
= H p Kh 
. 
dharm 
M-thermTh7-2.pm 5 
T 
T 
. = – 
dp p p 
h p p 
T 
T 
T 
2 523 
v 
H  G K J 
s K p K v s H G K J v 
p 
HG KJ  HG KJ H  − H G K G K J J ; . 
F I 
K H K G J 
s 
T H K 
GF  T JI
374 ENGINEERIN G THERMOD Y N AM ICS 
4. Entropy equation s (Td s equatio n s) : 
Tds = c dT + T 
F p I 
dv 
v 
Tds = cpdT – T H T K p 
dp 
5. Equatio n s for interna l energy and enthalpy : 
F F 
H H 
I I 
v K T K 
G u J = T G p J – p 
T v 
F p I du = c dT + Tv 
−pV dv 
SR T 
G p K = v – T G v J 
R U 
dh = cpdT + |v T 
T 
p| dp 
...(1) 
...(2) 
...(1) 
...[1 (a)] 
...(2) 
...[2 (a)] 
| | 
OBJECTIVE TYPE QUESTIONS 
Choose the Correct Answer : 
1. The specific heat at constan t pressure (c ) is given by 
(a) cp = T 
F s I 
p 
(b) cp = T 
F T I 
p 
(c) cp = T 
F 
T 
I 
p 
(d) cp = T 
F 
T 
I 
p 
. 
2. The specific heat relation is 
(a) (cp – cv) = 
vT2 
(c) (cp – cv) = 
pTK 
3. The relation of intern al energy is 
(a) du = 
F K 
cv 
I 
F I G J 
dp + H 
cp −pK dv 
(c) du = H  
cpJ dp + G v 
−vJ dv 
4. Tds equatio n is 
(a) Tds = cvdT + 
T 
dv 
(b) (cp – cv) = 
vTK 
(d) (cp – cv) = 
v T 
. 
F 
H K J v  
(b) du = G K 
c I dp + H 
cp pK dv 
GF K 
JI F HG cv I (d) du = H cpK dp + −pKJ 
v dv. 
 (b) Tds = cpdT – 
T 
dv 
(c) Tds = cvdT + 
TK 
dv (d) Tds = cvdT + 
T 
dp. 
Answers 
1. (a) 2. (a) 3. (a) 4. (a). 
dharm 
M-thermTh7-2.pm 5 
F I 
  
v H  T K 
U 
W 
F 
H 
I 
T 
F I 
H K p 
v − F  
H G I 
K S J V 
K  2 
 2 K 
H  K 
v HG  KJ 
T 
G v J 
G J 
 h J 
T 
T W 
p 
H H G K G K J J 
T s 
H H G K G K v J v J 
G J v  
GF K I 
K 
c F p I 
H K 
2 
v  
F I G J 
 
K K 
 K
THERMO D Y N A M IC RELATION S 375 
1. Define the co-efficien t of : 
(i) Volume expansion 
EXERCISES 
(ii ) Isothermal compressibil ity 
(iii ) Adiabatic compressibili ty. 
2. Derive the Maxwell relation s and explain their importance in thermo dy namics. 
3. Show that the equatio n of state of a substance may be written in the form 
dv 
= – Kdp + dT. 
v 
4. A substance has the volume expansivity and isothermal compressibi li ty : 
= 
T 
; K = p 
Find the equation of state. NAns. 
pv 
constantQ 
5. For a perfect gas, show that the difference in specific heats is 
cp – cv = 
T 
. 
6. For the followin g given differen tial equatio n s, 
du = Tds – pdv 
and dh = Tds + vdp 
prove that for perfect gas equation , 
G u J = 0 and 
T 
F 
H 
G h J = 0. 
T 
7. Using the cyclic equation, prove that 
HTKv 
= KT . 
8. Prove that the change in entropy is given by 
ds = 
cv L KT 
. dp  
M  
Q 
p O 
dv. 
9. Deduce the followin g thermo dy namic relation s : 
(i) 
Fh I 
= v – T G v J = – c 
FT I 
(ii ) 
Fu I 
= T 
F p I 
– p. 
T p h T v 
10. Show that for a Van der Waals gas 
cp – cv = 
1 −2a (v −b)2 / RTv3 . 
11. A gas obeys p(v – b) = RT, where b is positive constan t. Find the expression for the Joule-Thomson co-efficien 
t of this gas. Could this gas be cooled effectiv ely by throttlin g ? 
12. The pressure on the block of copper of 1 kg is increased from 10 bar to 1000 bar in a reversible process 
maintainin g the temperatu re constant at 15°C. Determin e : 
(i) Work done on the copper durin g the process (ii ) Change in entropy 
(iii ) The heat transfer (iv) Change in internal energy 
(v) (cp – cv) for this change of state. 
The followin g data may be assumed : 
Volume expansivity () = 5 × 10–5/K 
Isothermal compressibi lity (K) = 8.6 × 10–12 m2/N 
Specific volume (v) = 0.114 × 10–3 m3/kg 
[Ans. (i) – 4.9 J/kg ; (ii ) – 0.57 J/kg K ; (iii ) – 164 J/kg ; (iv) – 159.1 J/kg ; 9.5 J/kg K] 
dharm 
M-thermTh7-2.pm 5 
F I 
1 1 
L O 
M T P 
R 
H  p K  
I 
p K 
GF  p JI  
T 
c 
N  v 
P 
F 
H 
I 
H G J p  K K G H K J G J G J 
T p p  H  v K H  T 
K R
Cap 7 relaciones termodinamica

More Related Content

What's hot

Rates of convergence in M-Estimation with an example from current status data
Rates of convergence in M-Estimation with an example from current status dataRates of convergence in M-Estimation with an example from current status data
Rates of convergence in M-Estimation with an example from current status data
Lu Mao
 
Hawkinrad a source_notes ii _secured
Hawkinrad a source_notes ii _securedHawkinrad a source_notes ii _secured
Hawkinrad a source_notes ii _secured
foxtrot jp R
 
Dealinggreensfncsolft sqrd
Dealinggreensfncsolft  sqrdDealinggreensfncsolft  sqrd
Dealinggreensfncsolft sqrd
foxtrot jp R
 

What's hot (18)

article 1
article 1article 1
article 1
 
Mathandphysicspart6subpart1 draftbacc
Mathandphysicspart6subpart1 draftbaccMathandphysicspart6subpart1 draftbacc
Mathandphysicspart6subpart1 draftbacc
 
Black Hole Dynamics From Atmospheric Science
Black Hole Dynamics From Atmospheric ScienceBlack Hole Dynamics From Atmospheric Science
Black Hole Dynamics From Atmospheric Science
 
Hawkinrad a sourceasd
Hawkinrad a sourceasdHawkinrad a sourceasd
Hawkinrad a sourceasd
 
Rates of convergence in M-Estimation with an example from current status data
Rates of convergence in M-Estimation with an example from current status dataRates of convergence in M-Estimation with an example from current status data
Rates of convergence in M-Estimation with an example from current status data
 
Topic 8 kft 131
Topic 8 kft 131Topic 8 kft 131
Topic 8 kft 131
 
Ideal solution
Ideal solutionIdeal solution
Ideal solution
 
Introduction to the Keldysh non-equlibrium Green's function technique
Introduction to the Keldysh non-equlibrium Green's function techniqueIntroduction to the Keldysh non-equlibrium Green's function technique
Introduction to the Keldysh non-equlibrium Green's function technique
 
Hawkinrad a source_notes ii _secured
Hawkinrad a source_notes ii _securedHawkinrad a source_notes ii _secured
Hawkinrad a source_notes ii _secured
 
Heat Conduction Simulation with FDM
Heat Conduction Simulation with FDMHeat Conduction Simulation with FDM
Heat Conduction Simulation with FDM
 
Dealinggreensfncsolft sqrd
Dealinggreensfncsolft  sqrdDealinggreensfncsolft  sqrd
Dealinggreensfncsolft sqrd
 
Chern-Simons Theory
Chern-Simons TheoryChern-Simons Theory
Chern-Simons Theory
 
PART I.4 - Physical Mathematics
PART I.4 - Physical MathematicsPART I.4 - Physical Mathematics
PART I.4 - Physical Mathematics
 
Wasserstein GAN
Wasserstein GANWasserstein GAN
Wasserstein GAN
 
Sw2gr1 set a
Sw2gr1 set aSw2gr1 set a
Sw2gr1 set a
 
Topic 10 kft 131
Topic 10 kft 131Topic 10 kft 131
Topic 10 kft 131
 
PART I.2 - Physical Mathematics
PART I.2 - Physical MathematicsPART I.2 - Physical Mathematics
PART I.2 - Physical Mathematics
 
Solution i ph o 32
Solution i ph o 32Solution i ph o 32
Solution i ph o 32
 

Viewers also liked

Freedom Metals interview questions and answers
Freedom Metals interview questions and answersFreedom Metals interview questions and answers
Freedom Metals interview questions and answers
taylorlorr
 
Crack control of slabs design booklet
Crack control of slabs design bookletCrack control of slabs design booklet
Crack control of slabs design booklet
Abdullah Anjum
 
Importance pdhpe
Importance pdhpeImportance pdhpe
Importance pdhpe
lolfeg
 
Sponda yleisesitys 31032014
Sponda yleisesitys 31032014Sponda yleisesitys 31032014
Sponda yleisesitys 31032014
SpondaPlc
 

Viewers also liked (20)

Haystax Carbon for Insider Threat Management
Haystax Carbon for Insider Threat ManagementHaystax Carbon for Insider Threat Management
Haystax Carbon for Insider Threat Management
 
Freedom Metals interview questions and answers
Freedom Metals interview questions and answersFreedom Metals interview questions and answers
Freedom Metals interview questions and answers
 
Evaluation Q7
Evaluation Q7Evaluation Q7
Evaluation Q7
 
Crack control of slabs design booklet
Crack control of slabs design bookletCrack control of slabs design booklet
Crack control of slabs design booklet
 
Modelo ecologico y_modelo_integral_de_intervencion
Modelo ecologico y_modelo_integral_de_intervencionModelo ecologico y_modelo_integral_de_intervencion
Modelo ecologico y_modelo_integral_de_intervencion
 
Sponda Results presentation Q4 2015
Sponda Results presentation Q4 2015Sponda Results presentation Q4 2015
Sponda Results presentation Q4 2015
 
Shooting
Shooting Shooting
Shooting
 
Question 4
Question 4Question 4
Question 4
 
Ahuakate hezurretik abiatuz...
Ahuakate hezurretik abiatuz...Ahuakate hezurretik abiatuz...
Ahuakate hezurretik abiatuz...
 
Importance pdhpe
Importance pdhpeImportance pdhpe
Importance pdhpe
 
Guía de aprendizaje de inglés 3periodo
Guía de aprendizaje de inglés 3periodoGuía de aprendizaje de inglés 3periodo
Guía de aprendizaje de inglés 3periodo
 
Enterprise Threat Management
Enterprise Threat ManagementEnterprise Threat Management
Enterprise Threat Management
 
Sponda yleisesitys 31032014
Sponda yleisesitys 31032014Sponda yleisesitys 31032014
Sponda yleisesitys 31032014
 
Evolución de la web mapa
Evolución de la web mapaEvolución de la web mapa
Evolución de la web mapa
 
Haystax Technology Machine learning white paper
Haystax Technology Machine learning white paperHaystax Technology Machine learning white paper
Haystax Technology Machine learning white paper
 
Processing Events in Probabilistic Risk Assessment
Processing Events in Probabilistic Risk AssessmentProcessing Events in Probabilistic Risk Assessment
Processing Events in Probabilistic Risk Assessment
 
Major events
Major eventsMajor events
Major events
 
Critical assets protection
Critical assets protectionCritical assets protection
Critical assets protection
 
Literacy powerpointc
Literacy powerpointcLiteracy powerpointc
Literacy powerpointc
 
Overview of Haystax Technology
Overview of Haystax TechnologyOverview of Haystax Technology
Overview of Haystax Technology
 

Similar to Cap 7 relaciones termodinamica

Lets make a little excursion here to see what the thermodynamic co.pdf
Lets make a little excursion here to see what the thermodynamic co.pdfLets make a little excursion here to see what the thermodynamic co.pdf
Lets make a little excursion here to see what the thermodynamic co.pdf
anjanadistribution
 
Analysis and numerics of partial differential equations
Analysis and numerics of partial differential equationsAnalysis and numerics of partial differential equations
Analysis and numerics of partial differential equations
Springer
 
Proof of Fermat's Last Theorem (Using 6 Methods)
Proof of Fermat's Last Theorem (Using 6 Methods)Proof of Fermat's Last Theorem (Using 6 Methods)
Proof of Fermat's Last Theorem (Using 6 Methods)
nikos mantzakouras
 
On the-approximate-solution-of-a-nonlinear-singular-integral-equation
On the-approximate-solution-of-a-nonlinear-singular-integral-equationOn the-approximate-solution-of-a-nonlinear-singular-integral-equation
On the-approximate-solution-of-a-nonlinear-singular-integral-equation
Cemal Ardil
 

Similar to Cap 7 relaciones termodinamica (20)

Lets make a little excursion here to see what the thermodynamic co.pdf
Lets make a little excursion here to see what the thermodynamic co.pdfLets make a little excursion here to see what the thermodynamic co.pdf
Lets make a little excursion here to see what the thermodynamic co.pdf
 
Adomian Decomposition Method for Certain Space-Time Fractional Partial Differ...
Adomian Decomposition Method for Certain Space-Time Fractional Partial Differ...Adomian Decomposition Method for Certain Space-Time Fractional Partial Differ...
Adomian Decomposition Method for Certain Space-Time Fractional Partial Differ...
 
Spectral sum rules for conformal field theories
Spectral sum rules for conformal field theoriesSpectral sum rules for conformal field theories
Spectral sum rules for conformal field theories
 
Conformal field theories and three point functions
Conformal field theories and three point functionsConformal field theories and three point functions
Conformal field theories and three point functions
 
International Journal of Mathematics and Statistics Invention (IJMSI)
International Journal of Mathematics and Statistics Invention (IJMSI) International Journal of Mathematics and Statistics Invention (IJMSI)
International Journal of Mathematics and Statistics Invention (IJMSI)
 
Compiler Construction | Lecture 9 | Constraint Resolution
Compiler Construction | Lecture 9 | Constraint ResolutionCompiler Construction | Lecture 9 | Constraint Resolution
Compiler Construction | Lecture 9 | Constraint Resolution
 
Analysis and numerics of partial differential equations
Analysis and numerics of partial differential equationsAnalysis and numerics of partial differential equations
Analysis and numerics of partial differential equations
 
Thermodynamic ppt
Thermodynamic pptThermodynamic ppt
Thermodynamic ppt
 
Proof of Fermat's Last Theorem (Using 6 Methods)
Proof of Fermat's Last Theorem (Using 6 Methods)Proof of Fermat's Last Theorem (Using 6 Methods)
Proof of Fermat's Last Theorem (Using 6 Methods)
 
QMC Program: Trends and Advances in Monte Carlo Sampling Algorithms Workshop,...
QMC Program: Trends and Advances in Monte Carlo Sampling Algorithms Workshop,...QMC Program: Trends and Advances in Monte Carlo Sampling Algorithms Workshop,...
QMC Program: Trends and Advances in Monte Carlo Sampling Algorithms Workshop,...
 
Lect. 3 gibbs helmholtz equation, chemical potential, gibbs duhem equation
Lect. 3  gibbs helmholtz equation, chemical potential, gibbs duhem equationLect. 3  gibbs helmholtz equation, chemical potential, gibbs duhem equation
Lect. 3 gibbs helmholtz equation, chemical potential, gibbs duhem equation
 
On the dynamics of distillation processes
On the dynamics of distillation processesOn the dynamics of distillation processes
On the dynamics of distillation processes
 
Chap14_combined.ppt
Chap14_combined.pptChap14_combined.ppt
Chap14_combined.ppt
 
Function of a random variable lect5a.ppt
Function of a random variable lect5a.pptFunction of a random variable lect5a.ppt
Function of a random variable lect5a.ppt
 
Chap8_Sec3.ppt
Chap8_Sec3.pptChap8_Sec3.ppt
Chap8_Sec3.ppt
 
On the-approximate-solution-of-a-nonlinear-singular-integral-equation
On the-approximate-solution-of-a-nonlinear-singular-integral-equationOn the-approximate-solution-of-a-nonlinear-singular-integral-equation
On the-approximate-solution-of-a-nonlinear-singular-integral-equation
 
The lattice Boltzmann equation: background and boundary conditions
The lattice Boltzmann equation: background and boundary conditionsThe lattice Boltzmann equation: background and boundary conditions
The lattice Boltzmann equation: background and boundary conditions
 
cmftJYeZhuanTalk.pdf
cmftJYeZhuanTalk.pdfcmftJYeZhuanTalk.pdf
cmftJYeZhuanTalk.pdf
 
Ch06 6
Ch06 6Ch06 6
Ch06 6
 
SLC 2015 talk improved version
SLC 2015 talk improved versionSLC 2015 talk improved version
SLC 2015 talk improved version
 

Recently uploaded

Kuwait City MTP kit ((+919101817206)) Buy Abortion Pills Kuwait
Kuwait City MTP kit ((+919101817206)) Buy Abortion Pills KuwaitKuwait City MTP kit ((+919101817206)) Buy Abortion Pills Kuwait
Kuwait City MTP kit ((+919101817206)) Buy Abortion Pills Kuwait
jaanualu31
 
XXXXXXXXXXXXXXXXXXXXXXXXXXXXXXXXXXXXXXXXXXXXXXXXXXXX
XXXXXXXXXXXXXXXXXXXXXXXXXXXXXXXXXXXXXXXXXXXXXXXXXXXXXXXXXXXXXXXXXXXXXXXXXXXXXXXXXXXXXXXXXXXXXXXXXXXXXXXX
XXXXXXXXXXXXXXXXXXXXXXXXXXXXXXXXXXXXXXXXXXXXXXXXXXXX
ssuser89054b
 
Standard vs Custom Battery Packs - Decoding the Power Play
Standard vs Custom Battery Packs - Decoding the Power PlayStandard vs Custom Battery Packs - Decoding the Power Play
Standard vs Custom Battery Packs - Decoding the Power Play
Epec Engineered Technologies
 
Integrated Test Rig For HTFE-25 - Neometrix
Integrated Test Rig For HTFE-25 - NeometrixIntegrated Test Rig For HTFE-25 - Neometrix
Integrated Test Rig For HTFE-25 - Neometrix
Neometrix_Engineering_Pvt_Ltd
 
DeepFakes presentation : brief idea of DeepFakes
DeepFakes presentation : brief idea of DeepFakesDeepFakes presentation : brief idea of DeepFakes
DeepFakes presentation : brief idea of DeepFakes
MayuraD1
 

Recently uploaded (20)

S1S2 B.Arch MGU - HOA1&2 Module 3 -Temple Architecture of Kerala.pptx
S1S2 B.Arch MGU - HOA1&2 Module 3 -Temple Architecture of Kerala.pptxS1S2 B.Arch MGU - HOA1&2 Module 3 -Temple Architecture of Kerala.pptx
S1S2 B.Arch MGU - HOA1&2 Module 3 -Temple Architecture of Kerala.pptx
 
Kuwait City MTP kit ((+919101817206)) Buy Abortion Pills Kuwait
Kuwait City MTP kit ((+919101817206)) Buy Abortion Pills KuwaitKuwait City MTP kit ((+919101817206)) Buy Abortion Pills Kuwait
Kuwait City MTP kit ((+919101817206)) Buy Abortion Pills Kuwait
 
Hostel management system project report..pdf
Hostel management system project report..pdfHostel management system project report..pdf
Hostel management system project report..pdf
 
XXXXXXXXXXXXXXXXXXXXXXXXXXXXXXXXXXXXXXXXXXXXXXXXXXXX
XXXXXXXXXXXXXXXXXXXXXXXXXXXXXXXXXXXXXXXXXXXXXXXXXXXXXXXXXXXXXXXXXXXXXXXXXXXXXXXXXXXXXXXXXXXXXXXXXXXXXXXX
XXXXXXXXXXXXXXXXXXXXXXXXXXXXXXXXXXXXXXXXXXXXXXXXXXXX
 
Double Revolving field theory-how the rotor develops torque
Double Revolving field theory-how the rotor develops torqueDouble Revolving field theory-how the rotor develops torque
Double Revolving field theory-how the rotor develops torque
 
data_management_and _data_science_cheat_sheet.pdf
data_management_and _data_science_cheat_sheet.pdfdata_management_and _data_science_cheat_sheet.pdf
data_management_and _data_science_cheat_sheet.pdf
 
Bhubaneswar🌹Call Girls Bhubaneswar ❤Komal 9777949614 💟 Full Trusted CALL GIRL...
Bhubaneswar🌹Call Girls Bhubaneswar ❤Komal 9777949614 💟 Full Trusted CALL GIRL...Bhubaneswar🌹Call Girls Bhubaneswar ❤Komal 9777949614 💟 Full Trusted CALL GIRL...
Bhubaneswar🌹Call Girls Bhubaneswar ❤Komal 9777949614 💟 Full Trusted CALL GIRL...
 
Orlando’s Arnold Palmer Hospital Layout Strategy-1.pptx
Orlando’s Arnold Palmer Hospital Layout Strategy-1.pptxOrlando’s Arnold Palmer Hospital Layout Strategy-1.pptx
Orlando’s Arnold Palmer Hospital Layout Strategy-1.pptx
 
Standard vs Custom Battery Packs - Decoding the Power Play
Standard vs Custom Battery Packs - Decoding the Power PlayStandard vs Custom Battery Packs - Decoding the Power Play
Standard vs Custom Battery Packs - Decoding the Power Play
 
Block diagram reduction techniques in control systems.ppt
Block diagram reduction techniques in control systems.pptBlock diagram reduction techniques in control systems.ppt
Block diagram reduction techniques in control systems.ppt
 
Integrated Test Rig For HTFE-25 - Neometrix
Integrated Test Rig For HTFE-25 - NeometrixIntegrated Test Rig For HTFE-25 - Neometrix
Integrated Test Rig For HTFE-25 - Neometrix
 
School management system project Report.pdf
School management system project Report.pdfSchool management system project Report.pdf
School management system project Report.pdf
 
Wadi Rum luxhotel lodge Analysis case study.pptx
Wadi Rum luxhotel lodge Analysis case study.pptxWadi Rum luxhotel lodge Analysis case study.pptx
Wadi Rum luxhotel lodge Analysis case study.pptx
 
PE 459 LECTURE 2- natural gas basic concepts and properties
PE 459 LECTURE 2- natural gas basic concepts and propertiesPE 459 LECTURE 2- natural gas basic concepts and properties
PE 459 LECTURE 2- natural gas basic concepts and properties
 
HOA1&2 - Module 3 - PREHISTORCI ARCHITECTURE OF KERALA.pptx
HOA1&2 - Module 3 - PREHISTORCI ARCHITECTURE OF KERALA.pptxHOA1&2 - Module 3 - PREHISTORCI ARCHITECTURE OF KERALA.pptx
HOA1&2 - Module 3 - PREHISTORCI ARCHITECTURE OF KERALA.pptx
 
Design For Accessibility: Getting it right from the start
Design For Accessibility: Getting it right from the startDesign For Accessibility: Getting it right from the start
Design For Accessibility: Getting it right from the start
 
HAND TOOLS USED AT ELECTRONICS WORK PRESENTED BY KOUSTAV SARKAR
HAND TOOLS USED AT ELECTRONICS WORK PRESENTED BY KOUSTAV SARKARHAND TOOLS USED AT ELECTRONICS WORK PRESENTED BY KOUSTAV SARKAR
HAND TOOLS USED AT ELECTRONICS WORK PRESENTED BY KOUSTAV SARKAR
 
Online electricity billing project report..pdf
Online electricity billing project report..pdfOnline electricity billing project report..pdf
Online electricity billing project report..pdf
 
DeepFakes presentation : brief idea of DeepFakes
DeepFakes presentation : brief idea of DeepFakesDeepFakes presentation : brief idea of DeepFakes
DeepFakes presentation : brief idea of DeepFakes
 
Navigating Complexity: The Role of Trusted Partners and VIAS3D in Dassault Sy...
Navigating Complexity: The Role of Trusted Partners and VIAS3D in Dassault Sy...Navigating Complexity: The Role of Trusted Partners and VIAS3D in Dassault Sy...
Navigating Complexity: The Role of Trusted Partners and VIAS3D in Dassault Sy...
 

Cap 7 relaciones termodinamica

  • 1. 7 Thermodynamic Relations 7.1. General aspects. 7.2. Fundamen tals of partial differen tiation. 7.3. Some general thermodynam ic relations. 7.4. Entropy equation s (Tds equation s). 7.5. Equation s for internal energy and enthalp y . 7.6. Measurable quan tities : Equation of state, co-efficien t of expansion and compressibil i ty , speci fic heats, Joule-Thomso n co-efficien t 7.7. Clausiu s-Clap eryo n equatio n—Hig hligh t s — Objectiv e Type Questio n s—Exercises. 7.1. GENERAL ASPECTS In this chapter, some important thermodynamic relations are deduced ; principally those which are useful when tables of properties are to be compiled from limited experimental data, those which may be used when calculating the work and heat transfers associated with processes under-gone by a liquid or solid. It should be noted that the relations only apply to a substance in the solid phase when the stress, i.e. the pressure, is uniform in all directions ; if it is not, a single value for the pressure cannot be alloted to the system as a whole. Eight properties of a system, namely pressure (p), volume (v), temperature (T), internal energy (u), enthalpy (h), entropy (s), Helmholtz function (f) and Gibbs function (g) have been introduced in the previous chapters. h, f and g are sometimes referred to as thermodynami c potentials. Both f and g are useful when considering chemical reactions, and the former is of fundamental importance in statistical thermodynamics. The Gibbs function is also useful when considering processes involving a change of phase. Of the above eight properties only the first three, i.e., p, v and T are directly measurable. We shall find it convenient to introduce other combination of properties which are relatively easily measurable and which, together with measurements of p, v and T, enable the values of the remaining properties to be determined. These combinations of properties might be called ‘thermo-dynamic gradients’ ; they are all defined as the rate of change of one property with another while a third is kept constant. 7.2. FUNDAMENTALS OF PARTIAL DIFFERENTIATION Let three variables are represented by x, y and z. Their functional relationship may be expressed in the following forms : f(x, y, z) = 0 x = x(y, z) y = y(x, z) z = z(x, y) Let x is a function of two independent variables y and z x = x(y, z) ...(i) ...(ii) ...(iii) ...(iv) ...(7.1) 341
  • 2. 342 ENGINEERIN G THERMOD Y N AM ICS Then the differential of the dependent variable x is given by Fx I F x I y z z y ...(7.2) where dx is called an exact differential. F x I F I H K If G y J z = M and H z K y = N Then dx = Mdy + Ndz ...(7.3) Partial differentiation of M and N with respect to z and y, respectively, gives M 2 x z yz M N or z y and N 2 x y zy ...(7.4) dx is a perfect differential when eqn. (7.4) is satisfied for any function x. Similarly if y = y(x, z) and z = z(x, y) ...(7.5) then from these two relations, we have F y I F y dy = I x z dx + z x dz z z dz = x y dx + y x dy dy = G x I z dx + G z I F x x N I y F y dx  I x dy Q F y I F y I F z I F y I F z I L O M P M P H K G J H G K J = N x z z x x y Q dx + z x y x dy F y I F y I F z I L O M P M P = N x z z x x y Q dx + dy y y z  F F H H G G I I K K J J F H G I K J or x z x = 0 F yI F z I F y I or z x x y = – x x F z I F y I or y z x y z x = – 1 In terms of p, v and T, the following relation holds good FpI FT I F v I v T p v T p = – 1 ...(7.6) ...(7.7) ...(7.8) ...(7.9) dharm M-thermTh7-1.pm 5 dx = H K G J dy  H K G J dz  H G K J G H J K H G F J I G F I K H J K L O  H H G K G K J J H G K J  H H G K G K J J H G K J z x y z F H G I K J H K H K G J G J H K G J H K G J H K G J x G J   F y F y H K H K J J z z H K G J H G K M J P M P H K H K G J G J H G K J
  • 3. THERMO D Y N A M IC RELATION S 343 7.3. SOME GENERAL THERMODYNAMIC RELATIONS The first law applied to a closed system undergoing a reversible process states that dQ = du + pdv According to second law, dQ ds = rev. Combining these equations, we get Tds = du + pdv or du = Tds – pdv The properties h, f and g may also be put in terms of T, s, p and v as follows : dh = du + pdv + vdp = Tds + vdp Helmholtz free energy function, df = du – Tds – sdT = – pdv – sdT Gibb’s free energy function, dg = dh – Tds – sdT = vdp – sdT Each of these equations is a result of the two laws of thermodynamics. Since du, dh, df and dg are the exact differentials, we can express them as F uI F uI du = s v ds + v s dv, h h dh = s p ds + p dp, F f I F f I df = v T dv + T v dT, g dg = G pJ T g dp + G T J p dT. ...(7.10) ...(7.11) ...(7.12) ...(7.13) Comparing these equations with (7.10) to (7.13) we may equate the corresponding co-efficients. For example, from the two equations for du, we have F uI F uI s v = T and v s = – p The complete group of such relations may be summarised as follows : F uI F hI s = T = s F uI F f I = – p = v s v T g p s = v = p T f H T Kv = – s = G T J p ...(7.14) ...(7.15) ...(7.16) ...(7.17) dharm M-thermTh7-1.pm 5 T F H G I K J HG J K HG J K G F J I H K H F I K G J H H G K G K J J F I F I H H G K G K J J H G K J v H K G J p g F I F I F I s H K H K H G K J G H J K H G F h I J K G H K J G J H K
  • 4. 344 ENGINEERIN G THERMOD Y N AM ICS F T I F pI Also, HG K − HG K J J H K s H K v s F T I F vI p s p F p I F sI T v v T v s T p p T ...(7.18) ...(7.19) ...(7.20) ...(7.21) The equations (7.18) to (7.21) are known as Maxwell relations. It must be emphasised that eqns. (7.14) to (7.21) do not refer to a process, but simply expres s relations between properties which must be satisfied when any system is in a state of equilibrium. Each partial differential co-efficient can itself be regarded as a property of state. The state may be defined by a point on a three dimensional surface, the surface representing all possible states of stable equilibrium. 7.4. ENTROPY EQUATIONS (Tds Equations) Since entropy may be expressed as a function of any other two properties, e.g. temperature T and specific volume v, s = f(T, v) F s I F s I i.e., ds = T v dT + v T dv s s or Tds = T T v dT + T v T dv ...(7.22) But for a reversible constant volume change dq = cv (dT)v = T(ds)v s or cv = T T F s I F p I But, v T = T v ...(7.23) [Maxwell’s eqn. (7.20)] Hence, substituting in eqn. (7.22), we get Tds = cv dT + T H T Kv dv ...(7.24) This is known as the first form of entropy equation or the first Tds equation. Similarly, writing s = f(T, p) Tds = T H T K p dT + T H pKT dp ...(7.25) dharm M-thermTh7-1.pm 5 s v F H G I K J − F H G I K J v F H I K G J  G J H  H G K G K J J H H G K G K J J HF I HF I G K G K J J G J H H G K G K J J F p G I J F G s J I G F s J I
  • 5. THERMO D Y N A M IC RELATION S 345 where cp = T H T K p s v G J Also G pJ T = – H T K p ...(7.26) [Maxwell’s eqn. (7.21)] whence, substituting in eqn. (7.25) Tds = cpdT – T H T K p dp ...(7.27) This is known as the second form of entropy equation or the second Tds equation. 7.5. EQUATIONS FOR INTERNAL ENERGY AND ENTHALPY (i) Let F uI To evaluate v T Then or But Hence u = f(T, v) F uI F uI F u du = I T v dT + v T dv = cv dT + v T dv let u = f (s, v) F uI F uI du = s ds + v dv F uI FuI F s I F uI = v s v v F uI F s I F s I F uI = T, = s v v T T v v s = – p F uI F p I = T v T T v – p ...(7.28) ...(7.29) This is sometimes called the energy equation. From equation (7.28), we get | | | | du = cvdT + TT F p I v − pW dv (ii) To evaluate dh we can follow similar steps as under h = f(T, p) dh = F h I dT + G hI dp = cpdT + H p I T F H K J dp T ...(7.30) ...(7.31) dharm M-thermTh7-1.pm 5 F I F I H H H G K G K G K J J J v s H H G K G K J J T v T s H K H K H K H K H H H H G K G K G K G K J J J , J H H G K G K J J H  T K S G J V T p H K G J p GF s IJ H K F v I G J H G K J G J G J G J  G J R U F h G K J
  • 6. 346 FhI To find p T ; Then, But ENGINEERIN G THERMOD Y N AM ICS let h = f(s, p) F hI FhI dh = s ds + p dp FhI FhI F s I F hI p T = s p p T + p s F hI = T, G s J = – G vJ , G hJ = v p T p s F I K Hence G pJ T = v – T H T K p ...(7.32) From eqn. (7.31), we get dh = c p R U S V T| T W| dT + |v −T H v I p | dp ...(7.33) 7.6. MEASURABLE QUANTITIES Out of eight thermodynamic properties, as earlier stated, only p, v and T are directly measurable. Let us now examine the information that can be obtained from measurements of these primary properties, and then see what other easily measurable quantities can be introduced. The following will be discussed : (i) Equation of state (ii) Co-efficient of expansion and compressibility (iii) Specific heats (iv) Joule-Thomson co-efficient. 7.6.1. Equation of State Let us imagine a series of experiments in which the volume of a substance is measured over a range of temperatures while the pressure is maintained constant, this being repeated for various pressures. The results might be represented graphically by a three-dimens ional surface, or by a family of constant pressure lines on a v-T diagram. It is useful if an equation can be found to express the relation between p, v and T, and this can always be done over a limited range of states. No single equation will hold for all phases of a substance, and usually more than one equation is required even in one phase if the accuracy of the equation is to match that of the experimental results. Equations relating p, v and T are called equations of state or characteristic equations. Accurate equations of state are usually complicated, a typical form being pv = A + B  C + ...... where A, B, C, ...... are functions of temperature which differ for different substances. An equation of state of a particular substance is an empirical result, and it cannot be deduced from the laws of thermodynamics. Nevertheless the general form of the equation may be dharm M-thermTh7-1.pm 5 p H K G J s H K G J F I F I F I H G K J H HG JK HG JK H G K G K J J H s K G J H p K H p K H p K h H F G v J I F G K J v v 2
  • 7. THERMO D Y N A M IC RELATION S 347 predicted from hypotheses about the microscopic structure of matter. This type of prediction has been developed to a high degree of precision for gases, and to a lesser extent for liquids and solids. The simplest postulates about the molecular structure of gases lead to the concept of the perfect gas which has the equation of state pv = RT. Experiments have shown that the behaviour of real gases at low pressure with high temperature agrees well with this equation. 7.6.2. Co-efficient of Expansion and Compressibility From p-v-T measurements, we find that an equation of state is not the only useful informa-tion which can be obtained. When the experimental results are plotted as a series of constant pressure lines on a v-T diagrams, as in Fig. 7.1 (a), the slope of a constant pressure line at any v given state is T . If the gradient is divided by the volume at that state, we have a value of a property of the substance called its co-efficient of cubical expansion . That is, Fig. 7.1. Determinatio n of co-efficien t of expansion from p-v-T data. = v H T K p ...(7.34) Value of can be tabulated for a range of pressures and temperatures, or plotted graphically as in Fig. 7.2 (b). For solids and liquids over the normal working range of pressure and tempera-ture, the variation of  is small and can often be neglected. In tables of physical properties  is usually quoted as an average value over a small range of temperature, the pressure being atmos-pheric. This average co-efficient may be symbolised by  and it is defined by v2 −v v (T2 −T ) ...(7.35) Fig. 7.2 (a) can be replotted to show the variation of volume with pressure for various v constant values of temperature. In this case, the gradient of a curve at any state is p . When this gradient is divided by the volume at that state, we have a property known as the compressibility K of the substance. Since this gradient is always negative, i.e., the volume of a substance always decreases with increase of pressure when the temperature is constant, the compress ibility is usually made a positive quantity by defining it as dharm M-thermTh7-1.pm 5 = 1 F H G I K J p 1 GF  v IJ 1 1 F H G I K J T
  • 8. 348 ENGINEERIN G THERMOD Y N AM ICS Fig. 7.2. Determinatio n of compressibi lity from p-T data. K = – F I K 1 G vJ ...(7.36) T K can be regarded as a constant for many purposes for solids and liquids. In tables of properties it is often quoted as an average a value over a small range of pressure at atmospheric temperature, i.e., v2 −v v ( p2 − p ) When  and K are known, we have F p I F TI v T v v p p T v K = v and G vJ = – Kv, F p I  T v K Since H ...(7.37) When the equation of state is known, the co-efficient of cubical expansion and compressibility can be found by differentiation. For a perfect gas, for example, we have F T J = R v K T  G J 2 and H   RT v Hence = 1 F v K p = pv = T , and K = – v HpKT = RT = p . 7.6.3. Specific Heats Following are the three differential co-efficients which can be relatively easily determined experimentally. dharm M-thermTh7-1.pm 5 K = – 1 H K H K G J G J H G K J = – 1 F I F I H G K J = v H p 1 1 F I G J T p H K p T I F I H G K p p p p v H T G I J R 1 1 F v I G J p v 2 1
  • 9. THERMO D Y N A M IC RELATION S 349 F uI Consider the first quantity T . During a process at constant volume, the first law informs us that an increase of internal energy is equal to heat supplied. If a calorimetric experi-ment is conducted with a known mass of substance at constant volume, the quantity of heat Q required to raise the temperature of unit mass by ΔT may be measured. We can then write : H ΔT K= H ΔT Kv v . The quantity obtained this way is known as the mean specific heat at constant volume over the temperature range ΔT. It is found to vary with the conditions of the experiment, i.e., with the temperature range and the specific volume of the substance. As the temperature range is reduced the value approaches that of H T K , and the true specific heat at constant volume is defined by cv = G T K . This is a property of the substance and in general its value varies with the state of the substance, e.g., with temperature and pressure. According to first law of thermodynamics the heat supplied is equal to the increase of enthalpy during a reversible constant pressure process. Therefore, a calorimetric experiment carried out with a substance at constant pressure gives us, G ΔT J = G ΔT J p which is the mean specific heat at constant pressure. As the range of temperature is made infinites imally small, this becomes the rate of change of enthalpy with temperature at a particular state defined by T and p, and this is h true specific heat at constant pressure defined by c p = T . c p also varies with the state, e.g., with pressure and temperature. The description of experimental methods of determining c and c can be found in texts on physics. When solids and liquids are considered, it is not easy to measure c owing to the stresses set up when such a substance is prevented from expanding. However, a relation between c , c , and K can be found as follows, from which c may be obtained if the remaining three properties have been measured. The First Law of Thermodynamics, for a reversible process states that dQ = du + p dv Since we may write u = (T, v), we have du = H du K v dT + H u K T dv  dQ = H uJ v dT + Tp H v J T W dv = cv dT + Tp H v J T W dv This is true for any reversible process, and so, for a reversible constant pressure process, dQ = cp(dT)p = cv(dT)p + |p H u K | (dv)p Hence cp – cv = R |S F uJ | U G F V v I T| p W| H J K T p Also H p Kv = G s J T = 1 Sp H u KT V , and therefore cp – cv = T H T Kv H T K p dharm M-thermTh7-1.pm 5 H G K J v F Δu I F Q I G J G J GF u IJ F I v u H J v Δh p F H I K F Q H I K F H G I K J p p v v p v v F I F I  G J G J T v F I R| F I U| R| F I U| T u u G K G K G K |S |V |S |V R| F I U| T v S G J V T W H v G I K T F I R| F I U| G J T F H I K T| G J v T v W| G F p J I G F v J I
  • 10. 350 ENGINEERIN G THERMOD Y N AM ICS Now, from eqns. (7.34) and (7.37), we have cp – cv = 2Tv ...(7.38) Thus at any state defined by T and v, c can be found if c ,  and K are known for the substance at that state. The values of T, v and K are always positive and, although may some-times be negative (e.g., between 0° and 4°C water contracts on heating at constant pressure), 2 is always positive. It follows that cp is always greater than cv. The other expressions for cp and cv can be obtained by using the equation (7.14) as follows : Since cv = H u Kv = H u Kv H s Kv We have cv = T H T J ...(7.39) Similarly, cp = H T K p = F h s s I p F T I p Hence, cp = T H T K p ...(7.40) Alternative Expressions for Internal Energy and Enthalpy (i) Alternative expressions for equations (7.29) and (7.32) can be obtained as follows : G uI = T G p I – p ...(7.29) v F p I F T I F vI T v v p T or H T J v = – H T K p H v KT = + Kv = K Substituting in eqn. (7.29), we get H v KT = T K – p ...(7.41) F I G J Thus, du = cvdT + H T − pK dv ...[7.28 (a)] Similarly, H pJ = v – T H T K ...(7.32) F u I But by definition, T p = v Hence H pKT = v(1 – T) ...(7.42) dharm M-thermTh7-1.pm 5 K v p GF IJ GF IJ GF IJ T s T GF s I K v F G h J I H K H K G J G J F G s J I F F H J J v K H T K T But H H G K G K J J H K G J = – 1 GF I F I F I K p G J G J v p v  GF IJ u  K F G I K h T GF v IJ p G H J K F I G h J
  • 11. THERMO D Y N A M IC RELATION S 351 Thus (ii) Since or Hence dh = c p dT + v(1 – T) dp u = h – pv G uJ = F hI – p G vJ – v H p K HG p JK H p K T T T = v – vT + pKv – v G uJ = pKv – vT T ...[7.31 (a)] ...(7.43) 7.6.4. Joule-Thomson Co-efficient Let us consider the partial differential co-efficient H p K . We know that if a fluid is flowing through a pipe, and the pressure is reduced by a throttling process, the enthalpies on either side of the restriction may be equal. The throttling process is illustrated in Fig. 7.3 (a). The velocity increases at the restriction, with a consequent decrease of enthalpy, but this increase of kinetic energy is dissipated by friction, as the eddies die down after restriction. The steady-flow energy equation implies that the enthalpy of the fluid is restored to its initial value if the flow is adiabatic and if the velocity before restriction is equal to that downstream of it. These conditions are very nearly satisfied in the following experi-ment which is usually referred to as the Joule-Thomson experiment. T Constant h lines p1,T 1 p2, T2     Fluid      p2, T2 p1, T1 Slope =   p (a) (b) Fig. 7.3. Determination of Joule-Thomson co-efficien t. Through a porous plug (inserted in a pipe) a fluid is allowed to flow steadily from a high pressure to a low pressure. The pipe is well lagged so that any heat flow to or from the fluid is negligible when steady conditions have been reached. Furthermore, the velocity of the flow is kept low, and any difference between the kinetic energy upstream and downstream of the plug is negligible. A porous plug is used because the local increase of directional kinetic energy, caused by the restriction, is rapidly converted to random molecular energy by viscous friction in fine passages of the plug. Irregularities in the flow die out in a very short distance downstream of the plug, and dharm M-thermTh7-1.pm 5 F I F I F I H p K F G I J T h
  • 12. 352 ENGINEERIN G THERMOD Y N AM ICS temperature and pressure measurements taken there will be values for the fluid in a state of thermodynamic equilibrium. By keeping the upstream pressure and temperature constant at p and T , the downstream pressure p is reduced in steps and the corresponding temperature T is measured. The fluid in the successive states defined by the values of p and T must always have the same value of the enthalpy, namely the value of the enthalpy corresponding to the state defined by p and T . From these results, points representing equilibrium states of the same enthalpy can be plotted on a T-s diagram, and joined up to form a curve of constant enthalpy. The curve does not represent the throttling process itself, which is irreversible. During the actual process, the fluid undergoes first a decrease and then an increase of enthalpy, and no single value of the specific enthalpy can be ascribed to all elements of the fluid. If the experiment is repeated with different values of p and T , a family of curves may be obtained (covering a range of values of enthalpy) as shown in Fig. 7.3 (b). The slope of a curve [Fig. 7.3 (b)] at any point in the field is a function only of the state of the fluid, it is the Joule-Thomson co-efficient , defined by = G p J . The change of temperature due to a throttling process is small and, if the fluid is a gas, it may be an increase or decrease. At any particular pressure there is a temperature, the temperature of inversion, above which a gas can never be cooled by a throttling process. Both c and , as it may be seen, are defined in terms of p, T and h. The third partial differential co-efficient based on these three properties is given as follows : FhI F p I F T I p T h h p = – 1 F I h Hence p T = – cp ...(7.44) may be expressed in terms of cp, p, v and T as follows : The property relation for dh is dh = T ds + v dp From second T ds equation, we have F v I Tds = cp dT – T T dp  dh = c dT – L T F v I −v O dp p For a constant enthalpy process dh = 0. Therefore, | F v I | 0 = (cp dT)h + NT T p W Qh L M R U O M |S |V P P or (cp dT)h = MST G TJ −vV dpP h   = G T J h = M M T G H J P P cp N T K p −v Q ...(7.45) ...(7.46) For an ideal gas, pv = RT ; v = RT dharm M-thermTh7-1.pm 5 H G K J p NM QP v −T dp H K G J F I R| T U| W L N O Q F I L F I O 1 1 2 2 2 2 1 1 1 1 F T I H K h p H K G J H H G K G K T J J H K G J p H  T G K M J P p v M | H K | P H p K 1  v p
  • 13. THERMO D Y N A M IC RELATION S 353 or  F v I R v T p p T  = cp HT  T −vK = 0. Therefore, if an ideal gas is throttled, there will not be any change in temperature. or Let h = f(p, T) h F h I Then dh = p dp + T dT F h I But T p F I K = cp h  dh = H pKT dp + cp dT For throttling process, dh = 0 h p p T T h p 1 h p p T h p T is known as the constant temperature co-efficient. ...(7.47) ...(7.48) ...(7.49) 7.7. CLAUSIUS-CLAPERYON EQUATION Clausius-Claperyon equation is a relationship between the saturation pressure, tempera-ture, the enthalpy of evaporation, and the specific volume of the two phases involved. This equa-tion provides a basis for calculations of properties in a two-phase region. It gives the slope of a curve separating the two phases in the p-T diagram. p Critical point Liquid Vapour Solid Triple point Sublimat ion curve T Fig. 7.4. p-T diagram. dharm M-thermTh7-1.pm 5 H K G J = = F I H T H K p H K G J F I  0 = F I H K G J F H G I K J + c c = – F H G I K J F H G I K J 1 v G J G J G J G J
  • 14. 354 ENGINEERIN G THERMOD Y N AM ICS The Clausius-Claperyon equation can be derived in different ways. The method given below involves the use of the Maxwell relation [eqn. (7.20)] F I F I H H G K G K J J p s = T v v T Let us consider the change of state from saturated liquid to saturated vapour of a pure substance which takes place at constant temperature. During the evaporation, the pressure and temperature are independent of volume. F I dp sg −sf dT vg −vf  H G K J = where, s = Specific entropy of saturated vapour, g s= Specific entropy of saturated liquid, f v= Specific volume of saturated vapour, and g v= Specific volume of saturated liquid. f Also, sg – sf = sfg = hfg and vg – vf = vfg where sfg = Increase in specific entropy, T vfg = Increase in specific volume, and hfg = Latent heat added during evaporation at saturation temperature T.  dp sg −sf sfg hfg dT vg −vf vfg T .vfg ...(7.50) This is known as Clausius-Claperyon or Claperyon equation for evaporation of liquids. The derivative dT is the slope of vapour pressure versus temperature curve. Knowing this slope and the specific volume vg and vf from experimental data, we can determine the enthalpy of evaporation, (hg – hf) which is relatively difficult to measure accurately. Eqn. (7.50) is also valid for the change from a solid to liquid, and from solid to a vapour. At very low pressures, if we assume v − v and the equation of the vapour is taken as pv = RT, then eqn. (7.50) becomes dp hfg hfg p dT Tvg RT 2 ...(7.51) or hfg = RT2 dp p dT ...(7.52) Eqn. (7.52) may be used to obtain the enthalpy of vapourisation. This equation can be rearranged as follows : dp = hfg . dT Integrating the above equation, we get dp = hfg dT h L T O p R NT T2 Q ...(7.53) dharm    = = p R T 2 p z R 2 z dp ~ g fg ln p 2 fg 1 1  M − P 1 1
  • 16. THERMO D Y N A M IC RELATION S 355 Knowing the vapour pressure p at temperature T we can find the vapour pressure p 1 1 2 corresponding to temperature T2 from eqn. (7.53). From eqn. (7.50), we see that the slope of the vapour pressure curve is always +ve, since v > v and h is always +ve. Consequently, the vapour pressure of any simple compressible substance increases with temperature. — It can be shown that the slope of the sublimation curve is also +ve for any pure substance. — However, the slope of the melting curve could be +ve or –ve. — For a substance that contracts on freezing, such as water, the slope of the melting curve will be negative. +Example 7.1. For a perfect gas, show that c p – c v = Np H v K T Q H TK p  pv v H v K T where  is the co-efficient of cubical/volume expansion. Solution. The first law of thermodynamics applied to a closed system undergoing a reversible process states as follows : dQ = du + pdv As per second law of thermodynamics, FdQI ds = rev. Combining these equations (i) and (ii), we have Tds = du + pdv ...(i) ...(ii) Also, since  Thus, h = u + pv dh = du + pdv + vdp = Tds + vdp Tds = du + pdv = dh – vdp Now, writing relation for u taking T and v as independent, we have du = F u I v dT + F v I T dv u = cv dT + u H G K J v T dv Similarly, writing relation for h taking T and p as independent, we have F h I FhI dh = T p dT + p dp FhI = cp dT + p T dp In the equation for Tds, substituting the value of du and dh, we have cv dT + F uJ T dv + pdv = cp dT + H pKT dp – vdp or c v dT + Mp FuI N H v K Q N H p K T P dv = c p dT – Lv −FhI T O dp dharm M-thermTh7-1.pm 5 H T G K J g f fg L F I O F I F I M P u  v u G J G J G J H K G J T H G K J F I H G K J H K G J T H K G J H G I v K F h I L O M Q P
  • 17. 356 ENGINEERIN G THERMOD Y N AM ICS Since the above equation is true for any process, therefore, it will also be true for the case when dp = 0 and hence (c p – c v ) (dT) p = Np HuKT Q (dv) p or (cp – cp) = Mp H v K P HTK p By definition, = 1 F v I p  The above equation becomes, cp – cv = Mp H v K P v or = pv+ v F v I T Proved. u +Example 7.2. Find the value of co-efficient of volume expansion  and isothermal compressibility K for a Van der Waals’ gas obeying Hp  v2 K (v −b) = RT. Solution. Van der Waals equation is Hp  v2 K (v −b) = RT Rearranging this equation, we can write RT a v −b v2 Now for we require HTK p . This can be found by writing the cyclic relation, H v K p HTKv HpKT = – 1 Hence  p T v p = – H vKT From the Van der Waals equation, F p I R T v v −b Also H v KT = – (v −b)2 + v3 Hence = 1 F v I = 1 M – G H J  GF  IJ p Kv P p N H v K T Q dharm M-thermTh7-1.pm 5 L F I O M v P L F I O F I T u N Q v v H  T K L O T F u I N Q H K F a I F a I p = − F u I F I F I F I T p v F I F I H K H T K F  p I v H K = F I p RT 2 a L F I O v H T K v T p M M M P P P
  • 18. THERMO D Y N A M IC RELATION S 357 or = 1 M − P P P v −b P . Rv 2 (v −b) . (Ans.) RTv 2a(v b) (v −b)2 v3 L O Also, K = – 1 H v K = – v M 2a 1 RT P = RTv 2 (v a v 2 b 2 . (Ans.) v3 (v −b)2 M M M M P M P Example 7.3. Prove that the internal energy of an ideal gas is a function of temperature alone. Solution. The equation of state for an ideal gas is given by p = RT But H v KT = T HT Kv − p [Eqn. (7.29)] = T v − p = p – p = 0. Thus, if the temperature remains constant, there is no change in internal energy with volume (and therefore also with pressure). Hence internal energy (u) is a function of temperature (T) alone. ...Proved. Example 7.4. Prove that specific heat at constant volume (c ) of a Van der Waals’ gas is a function of temperature alone. Solution. The Van der Waals equation of state is given by, RT a v −b v2 or H p Kv = v −b or G 2p J = 0 v Now Fdc I = T H p K  T Hence T H K G J  G J F v I H K v G J T = 0 v Thus c of a Van der Waals gas is independent of volume (and therefore of pressure also). Hence it is a function of temperature alone. +Example 7.5. Determine the following when a gas obeys Van der Waals’ equation, Hp  v2 K (v – b) = RT (i) Change in internal energy ; (ii) Change in enthalpy ; (iii) Change in entropy. Solution. (i) Change in internal energy : The change in internal energy is given by du = cvdT + MT HTKv − pP dv dharm M-thermTh7-1.pm 5 v R − RT 2a  L N O Q 3 2 − − F  I 1 M P v − b v p T − N Q 3 ) − 2 ( − ) v F u I F  p I R v p = − F I T R F I H T 2 K F I dv 2 2 c v v F a I L F  p I O N Q
  • 19. 358 But,              ENGINEERIN G THERMOD Y N AM ICS F p I L  R RT a UO T v = = R T v −b v2 v −b z 2 z 2 z 2 L F G R I J du c v dT  N T H − K Q M M P − p P NM T W QP O dv = cv z2 dT z2 L 1 1 1 T F v −b I − R v −b − O dv a V = cv 2 dT  2 M U MP 1 v −b 2 R RT H K v G J S RT RT − v −b  a P dv 2 1 1 z 2 z 2 L O N Q c dT .dv 1 1 u 2 – u 1 = c v (T 2 – T 1 ) + a F 1 − 1 I . (Ans.) 1 2 (ii) Change in enthalpy : The change in enthalpy is given by dh = c dT + Mv −T F v I P dp p FhI = 0 + v – T F v I ...(1) T p Let us consider p = f(v, T)  dp = H v KT dv + HT Kv dT  (dp)T = H v KT dv + 0 as dT = 0 ...(2) From equation (1), (dh)T = Nv −T HT K pQ (dp)T. O H Substituting the value (dh) of (dp) = v from −T Heqn. K (2), we Kget T NT v p T dv F I F I M p v  p P M P = Nv H vKT −T HT K p H v KT Q dv ...(3) Using the cyclic relation for p, v, T which is F v I FTI FpI T p p v v T  F v I G pJ −G p J p T v dharm M-thermTh7-1.pm 5 H K TS − WV N Q M P v v b v a v z  z v v G J = v 2 H K L O p NM H T K QP H p K H  T K F p I F p I F p I L F v I O M P M P T L M M v P p P Q  L F I F I F I O H K H H G K G K J J − 1 F I F I H T K H v K H T K
  • 20. THERMO D Y N A M IC RELATION S 359 Substituting this value in eqn. (3), we get (dh) = Mv FpI T F p I P dv ...(4) T v For Van der Waals equation p  LF RT I a O T v v −b v2 T = – (v −b)2  v3 ...(5) p R HT Kv = M P T MH RT − a K Pv = v −b ...(6) R − Substituting the values of eqns. (5) and (6) in equation (dh) (1), we get T = Mv (v −b)2  v aU T H v R bK P dv  2 2 v 1 (dh)T = – RT 1 (v −b)2 dv + 2a F I G J 2 2 dv 1 dv + RT 1 (v −b) (h2 – h1)T = – RT Mloge F v −bJ −b R v 2 G − I H K TS 1 − 1 v −b v −b WV UP – 2a F 1 − 1 I RT log F v2 −b I 2 1 1 = bRT M(v2 −b) − (v1 −b) Q – 2a M v2 − v1 P . (Ans.) (iii) Change in entropy : The change in entropy is given by ds = cp dT  H p Kv . dv For Van der Waals equation, p R T v v −b ...as per eqn. (6)  ds = cv T  v −b dv 2  ds = cv 2 MdT P R 2 (v dv b)  s2 – s1 = cv loge M 1 P + R loge N v2 −b Q . (Ans.) Example 7.6. The equation of state in the given range of pressure and temperature is given by v = RT − C where C is constant. Derive an expression for change of enthalpy and entropy for this substance during an isothermal process. dharm M-thermTh7-1.pm 5 F I F I L O T N H  v K H  T K Q F I H v K H K = − N Q M RT 2a P F I  L F I O N G J v − b 2 v Q S RT T V W − L NM O QP 3 2 z z z z v2 b 1 2 1 L N O Q v v v b e H K H − K G J G J L 1 1 O L O N P 1 1 N Q T T H K  dT R z z L O z 1 1 N T Q 1 − L T O L O T 2 N Q v 1 −b M P p T 3
  • 21. 360 ENGINEERIN G THERMOD Y N AM ICS Solution. The general equation for finding dh is given by dh = cp dT + L v −T F M T P M P I p O dp 2 dh = 0 + z2 MR | T| v −T F T I p UP T as dT = 0 for isothermal change. From the given equation of state, we have HTK = R  3C ...(i)  h2 – h1 = z2 L M O N Q RF RT − C I − | RT − 3C U dpPT = M 2 1 | T| V G − 4C J dpP T  − 4C [( p2 − p )] The general equation for finding ds is given by ds = cp dT −H v K p dp z2 ds = L − 2F M M T Q I p dp O T as dT = 0 for isothermal change. (s2 – s1) = M 2 Substituting the value from eqn. (i), we get −G R  3C I dp O H p1 K G J H T 4 K 4 1 = – R loge Fp2 I – F3CI (p2 – p1) (Ans.) Example 7.7. For a perfect gas obeying pv = RT, show that c and c are independent of pressure. Solution. Let s = f(T, v) Then ds = HT Kv dT + Hv KT dv Also u = f(T, v) Then du = HT Kv dT + H v KT dv = cv dT + H vKT dv Also, du = Tds – pdv Tds – pdv = cv dT + HuKT dv ds = cv T  T MF v I T  pP dv dharm M-thermTh7-1.pm 5 v N H K Q z L O 1 v H G K S J V N W| Q 1 M P F v I p p T 4 H p T p T G K S J | W| M P 3 3 1 N L z H F K T 3 I Q O 1 T 3 T F I T T 1 H G K J N z v P 1 P L F z H K J N Q P p T T v p F s I F s I F u I F u I F u I F I v dT L u O N H  K Q 1
  • 22. THERMO D Y N A M IC RELATION S 361 Equating the co-efficients of dT in the two equations of ds, we have F I  H G K J cv s T T v c v = T H s K Fcv I 2 s v T Tv From eqn. (7.20), F s I F p I v T T v 2 s 2 p vT T2 Fcv I F 2p I v T T2 v  H G K J Also p = RT ...(Given) F p I R T v F p I T v F I H K 0 or G  v J T 0 2 c v This shows that cv is a function of T alone, or cv is independent of pressure. Also, cp = T H s K p H p K T Tp From eqn. (7.21), F sI −F v I T p 2s 2v pT T2 p G cp J −T G 2 2 J T p Again, v = R F v I R T p F 2v I and T2 p = 0 ; ...(Given) H p KT = 0 This shows that cp is a function of T alone or cp is independent of pressure. dharm M-thermTh7-2.pm 5 p H K G J  H G K J F I T v H K G J  T H K  H K F I v H K G J H G K J T v H K  v  H G K J 2  F I T F  I G J c p  s T 2 H p K H T K F I − H K G J F I F I v H p K H  T K p F G c I p J
  • 23. 362 ENGINEERIN G THERMOD Y N AM ICS Example 7.8. Using the first Maxwell equation, derive the remaining three. Solution. The first Maxwell relation is as follows : F T I F  p I H v Ks  −H s Kv (1) Using the cyclic relation T v s H v Ks . H s KT . HT Kv = – 1 s T s  Hv KT = – H v Ks . HT Kv Substituting the value from eqn. (i) in eqn. (ii), we get s p s Hv KT = H s Kv . HT Kv Using the chain rule, p s T H s Kv . HT Kv . H p Kv = 1 Substituting the value of eqn. (iv) in eqn. (iii), we get F s I F p I Hv KT = HT Kv This is Maxwell Third relation. (2) Again using the cyclic relation vI F H s I F I F K H p K H K p v . . = – 1 v s s p v FpI F v I s  H K = – H K H K . p s v p s Substituting the value from eqn. (i) into eqn. (v) v T v HsK p = H v Ks . HpKs Again using the chain rule, T v p H v Ks . HpKs . HT Ks = 1 Substituting the value of (vi) into (v), we get F v I F  T I HsK p H p Ks This is Maxwell second relation. (3) HTK p . H p Kv . H vKT = – 1 ...(i) (Eqn. 7.18) ...(ii) ...(iii) ...(iv) ...(v) ...(vi ) dharm F I F I F I F I F I F I F I F I F I F I F I F I F I F I F I F I F I F I F I F I F I F I v T p
  • 25. THERMO D Y N A M IC RELATION S 363 F v I F p I F v I T H K = – H K H K . p T v p T F p I F s I F s I F v I = – H s Kv HT Kv HpKT H s KT Substituting the value from eqn. (i), we get F v I F TI F s I F s I F vI p v s T v p H T K = G H K J G H K J G H J K T H G K J s T RS F T I F v I F s I UV F s I F s I = TH v Ks . H sKT . HTKvW H pKT = – HpKT F I  H v K p = – HpKT This is Maxwell fourth relation. Example 7.9. Derive the following relations : (i) u = a – T F a T I (ii) h = g – T F g I p (iii) cv = – T G 2a J (iv) cp = – T F 2 g J G K 2 T T H  v p H  K 2 where a = Helmholtz function (per unit mass), and g = Gibbs function (per unit mass). Solution. (i) Let a = f(v, T) Then da = H v KT dv + HT Kv dT Also da = – pdv – sdT Comparing the co-efficients of dT, we get HT Kv = – s Also or Hence (ii) Let Then Also a = u – Ts u = a + Ts = a – T HT Kv u = a – T HT Kv . (Ans.) g = f(p, T) dg = HpKT dp + HTK p dT dg = vdp – sdT Comparing the co-efficients of dT, we get HTK p = – s dharm M-thermTh7-2.pm 5 F I F I F I F I  T F s I H K v H  T K F I I F a I F a I F a I a a g g F g I
  • 26. 364 ENGINEERIN G THERMOD Y N AM ICS Also h = g + Ts = g – T HTK p Hence h = g – T HTK p . (Ans.) (iii) From eqn. (7.23), we have c v = T H s K Also F a I = – s v or HT Kv 2a = – HT2 Kv From eqns. (i) and (ii), we get c v 2a = – T G T2 J v . (Ans.) (iv) From eqn. (7.26), we have cp = T H s K p Also HTK = – s or F T I p s G J T 2 = – H 2g K p From eqns. (i) and (ii), we get 2 g cp = – T HT2 K p . (Ans.) ...(i) ...(ii) ...(i) ...(ii) Example 7.10. Find the expression for ds in terms of dT and dp. Solution. Let s = f(T, p) Then ds = H s K p . dT + HpKT dp As per Maxwell relation (7.21) HpKT = – HTK p Substituting this in the above equation, we get ds = H s K p dT – H v K p . dp ...(i) The enthalpy is given by dh = cpdT = Tds + vdp Dividing by dT at constant pressure HTK p = cp = T HTK p + 0 (as dp = 0 when pressure is constant) dharm M-thermTh7-2.pm 5 F I H T K F I F I F I F I F I p F I F I F g I F g I T v s G J H K T g H K G J F I F s I  T F s I F v I F I F I T  T F h I F s I
  • 27. THERMO D Y N A M IC RELATION S 365 Now substituting this in eqn. (i), we get But ds = cp dT F I −H s K . dp ...(ii) 1F v I v T p Substituting this in eqn. (ii), we get ds = cp dT – vdp (Ans.) T Example 7.11. Derive the following relations : F T I F T I (i) H p K = Tv (ii) H v K = – T c K . where = Co-efficient of cubical expansion, and K = Isothermal compressibility. Solution. (i) Using the Maxwell relation F (7.19), we have T I FvI F v I FTI s s p T p s p Also cp = T HTK From eqn. (7.34), = v F T I p H p Ks  vT i.e., H p Ks  c  . (Ans.) (ii) Using the Maxwell relation (7.18) FT I FpI F p I FT I s s v T v s v Also cv = T HT K K = – 1 F v I FTI T F p I v c T Also FpI F v I FTI = – 1 T p v i.e., H p Kv = – HvKT HTK p = – H−vK K v = K (Eqn. 7.23) (Eqn. 7.36) dharm M-thermTh7-2.pm 5 p G J T T = H K H p K = H  K = H K H K F I F I F I F I v H  p K s v v H K H K F I F I F I F I c p s s v s p 1 v H K  T c p  T Tv p H v K = – H s K = – H K H K v T Then = – H v K H T K H p K T p v 1 
  • 28. 366 ENGINEERIN G THERMOD Y N AM ICS F I v c K v  HT Ks = −T . (Ans.) +Example 7.12. Derive the third Tds equation F T I Tds = cv H p Kv dp + cp F v I p dv and also show that this may be written as : Solution. Let Then or But Hence Also and Tds = cv Kdp + c v dv. dss == f(p, v) F p T H G K J I dp + HsK dv s Tds = T F s I dp + T H s K dv v G J p F I F I T p T v = T H s Kv HTKv F I + T F s I p FTI p dv H s Kv = T and H s K p = T Tds = cvHpK dp cp HvK dv ...Proved. F I F I F I p = FpI − F v I = – H v K F p I = K p H K H K v T 1 T T p H v K p = v Substituting these values in the above Tds equation, we get Tds = c  K dp  v dv ...Proved. Example 7.13. Using Maxwell relation derive the following Tds equation Tds = cp dT – T HTK p dp. (U.P.S.C. 1988) Solution. where cp = T H TK p Also, s = f (T, p) Tds = T HTK p dT + T HpKT dp H s KT = – F v I p ...(i) ......Maxwell relation Substituting these in eqn. (i), we get Tds = cp dT – T HTK p dp. (Ans.) dharm  p H K v F I p H K v  v p H K H K F I v p F I F I F I F I F I T c v  T c p T T T v H K H K v T  T 1 c v p F v I GF s IJ s s F I p H T K F v I
  • 30. THERMO D Y N A M IC RELATION S 367 Example 7.14. Derive the following relations F I FT I TG p J − p H K u v Solution. FT I H  K T c v = . v can be expressed as follows : u H v K = F vI F I F I Kv = – F uI T u u v T − H H K H K H K u T T v H v K F T I u Also Tds = du + pdv or du = Tds – pdv or HuK = T H s K – p H v K or v F uI T v T = T F s I – p T T or H u Kv = T H s Kv F I T T Dividing eqn. (i) by eqn. (ii), we get  F I K F I FT I THvK − p v H K = s T u s T v s Also cv = T HT K and F s H G K J v I T = F p H K F H T I v u T ... Maxwell relation Substituting these value in eqn. (iii), we get TG p J − p v u = cv v ...Proved. +Example 7.15. Prove that for any fluid (i) HhKT = v HpKT + T H p Kv (ii) HhKT = v – T HTK p Show that for a fluid obeying van der Waal’s equation p = v −b – v2 ...(i) ...(ii) ...(iii) where R, a and b are constants h (enthalpy) = RTb − 2a + f(T) where f(T) is arbitrary. dharm F I F I F I H v K H v K I I T H v F I H K T  T I K F I F I F I F I F I v v T p v RT a v −b v
  • 32. 368 ENGINEERIN G THERMOD Y N AM ICS Solution. We know that ds = c v  p T dT  HT K dv Also dh = Tds + vdp = T M v dT F p I dvP + vdp v i.e., dh = c v dT + T H p Kv + dv + vdp Putting dT = 0, we get G hJ T G p J v G pJ ...Proved. T v T (ii) HhKT hKT H v KT H  = NT F    p I v vF pI vJ T Q G  i.e., h GF IJ G J T p F p I T = T H p K F vI + v Also F p I v H vJ T = – H v K p  Eqn. (i) becomes HpKT = v – T H vKp ...Proved. [Eqn. (7.24)] ...(i) Now and         i.e., RT a v −b v2 p RT 2a (v −b)2 v3 F p I R T v −b FhI L −RT 2aO F R I T (v −b)2 v3 v −b RTv 2a RT −RTv RT 2a 2 (v −b ) v 2 v −b (v −b)2 v −b v2 −RTv RT(v −b) 2a −RTv RTv −RTb 2a (v −b)2 v2 (v −b)2 v2 FhI −RTb 2a v T (v −b)2 v2 or RTb 2a v −b v This shows h depends on T and v. ...Proved. Example 7.16. Derive the following relations : (i) FhI = v – T v FTI FuI H p = – c(ii) = T K – p T p h T dharm M-thermTh7-2.pm 5 v F I L N O Q F I F I F I F I F I F I F I L O F I H K v T F I F I F I F I p = − F I H v K = –  H K v = N Q H K =  =  H K =  h = −  f(T) c T H  T K T H  v K H T K H  v K  p v p M G H J K G H J K P T v H p K T H K H K G T J G p K  T  h  T T H v K = v M  P T = –   =   F I F I H T K H K p H K H K p p v T v
  • 33. THERMO D Y N A M IC RELATION S 369 With the aid of eqn. (ii) show that F u I F v I F I H pKT = – T HTK – p HpKT The quantity cp p FTI is known as Joule-Thomson cooling effect. Show that this cooling h effect for a gas obeying the equation of state (v – b) = RT – C is equal to H 3C K −b. Solution. We know that F p I = – cp Also = cp L HT K p −v O  FhI = – T F v I −v O = v – T F v I T p p Also = H p Kh  H pKT = – cp H p Kh . ...[Eqn. (7.44)] ...[Eqn. (7.46)] ... Proved. (ii) Let Also u = f(T, v) du = HT Kv dT + H v KT dv = c v dT + H v KT dv ...(i) du = Tds – pdv Substituting the value of Tds [from eqn. 7.24], we get du = c v dT + T F p I G J v N  T Q dv – pdv = cv dT + MT H p Kv − pP dv ...(ii) From (i) and (ii), we get HuK = T HT K – p ...Proved. FuI FuI F v I p T v T p T or F p I u v = HpKp T T GF IJ T HTKv − pQ or H pKT = T F H G J K G H J F I T K H K I v F p I T – p G pJ T ...Proved. dharm M-thermTh7-2.pm 5 H K T NM QP L NM QP F I F I F I F I F I F I v H p K F I p T 2 T G 2 J h T 1 M F  v I P H p K H  T K M P H T K T h T u u u H K G J T L F I O F I F I v T G p J v Also H K = H K H K G J G J F I L F I O H K G J G J G  J N M P u  p  v v
  • 34. 370 We know that or Also and FpI F v I FTI v H K H K H K = – 1 T T p p v I K J − F F p v v T p T F p I H K H K G J G J u v T = – T F v T I p – p F p I T = 1 L O M P N Q c T p H  K M F uI p −vP ENGINEERIN G THERMOD Y N AM ICS ...Already proved. ...[Eqn. (7.46)] Now v – b = RT − C ...[Given] HTK p = R  2C Substituting this value in the expression of above, we get = 1 L T F H G I J N R  2C K −v O M Q or cp = T G R H  p 3 K T p T 2 2CJ – RT  C – b = 3 2 −b or cp H p Kh = T2 – b ...Proved. Example 7.17. The pressure on the block of copper of 1 kg is increased from 20 bar to 800 bar in a reversible process maintaining the temperature constant at 15°C. Determine the following : (i) Work done on the copper during the process, (ii) Change in entropy, (iii) The heat transfer, (iv) Change in internal energy, and (v) (cp – cv) for this change of state. Given :  (Volume expansitivity = 5 × 10–5/K, K (thermal compressibility) = 8.6 × 10 12 m2/N and v (specific volume) = 0.114 × 10–3 m3/kg. Solution. (i) Work done on the copper, W : Work done during isothermal compression is given by 2 W = pdv 1 The isothermal compressibility is given by K = – 1 F v I T  dv = – K(v.dp)T 2 2  W = – pKv.dp = – vK pdp 1 1 Since v and K remain essentially constant  W = – vK (p2 2 – p1 2) = – 0.114 10−3 8.6 10−12 [(800 × 105)2 – (20 × 105)2] dharm M-thermTh7-2.pm 5 F H G H H G G I I K K J J v T p H K T p T 2 F v I p T 3 c p 3 p T P F I C T GF IJ T 3 C – z v H  p K z z 2 2
  • 35. THERMO D Y N A M IC RELATION S 371 = – 0.114 8.610−15 × 1010 [(800)2 – (20)2] = – 0.114 8.6 10−5 (640000 – 400) = – 3.135 J/kg. (Ans.) The negative sign indicates that the work is done on the copper block. (ii) Change in entropy : The change in entropy can be found by using the following Maxwell relation : G pJ T = – H v K p F I = – F I v F T I p = – vas 1 H v K p =    (ds) T = – v (dp) T Integrating the above equation, assuming v and  remaining constant, we get s2 – s1 = – v (p2 – p1)T = – 0.114 × 10–3 × 5 × 10–5 [800 × 105 – 20 × 105] = – 0.114 × 10–3 × 5 (800 – 20) = – 0.446 J/kg K. (Ans.) (iii) The heat transfer, Q : For a reversible isothermal process, the heat transfer is given by : Q = T(s2 – s1) = (15 + 273)(– 0.4446) = – 128 J/kg. (Ans.) (iv) Change in internal energy, du : The change in internal energy is given by : du = Q – W = – 128 – (– 3.135) = – 124.8 J/kg. (Ans.) (v) cp – cv : The difference between the specific heat is given by : c p – c v = 2Tv ... [Eqn. (7.38)] (5 10−5)2 (15 273)  0.114 10− 3 = 8.6 10−12 = 9.54 J/kg K. (Ans.) Example 7.18. Using Clausius-Claperyon’s equation, estimate the enthalpy of vapourisation. The following data is given : At 200°C : vg = 0.1274 m3/kg ; vf = 0.001157 m3/kg ; H dT K = 32 kPa/K. Solution. Using the equation F dp I fg dT T (vg −v ) where, hfg = Enthalpy of vapourisation. Substituting the various values, we get 32 × 103 = (200 273)(0.1274 −0.001157)  hfg = 32 × 103 (200 + 273)(0.1274 – 0.001157) J = 1910.8 × 103 J/kg = 1910.8 kJ/kg. (Ans.) dharm M-thermTh7-2.pm 5 2 2 F I H K s v v T H K v T K F dp I G J H G K J = h s f h fg
  • 36. 372 ENGINEERIN G THERMOD Y N AM ICS Example 7.19. An ice skate is able to glide over the ice because the skate blade exerts sufficient pressure on the ice that a thin layer of ice is melted. The skate blade then glides over this thin melted water layer. Determine the pressure an ice skate blade must exert to allow smooth ice skate at – 10°C. The following data is given for the range of temperatures and pressures involved : hfg( ice) = 334 kJ/kg ; vliq. = 1 × 10 m3/kg ; vice = 1.01 × 103 m3/kg. Solution. Since it is a problem of phase change from solid to liquid, therefore, we can use Clausius-Claperyon equation given below : dp fg 1 dT fg T Multiplying both the sides by dT and integrating, we get p2 dp fg T2 dT 1 v g 1 T or (p2 – p1) = fg loge GT2 J ...(i) But at p1 = 1 atm., t1 = 0°C Thus, p1 = 1.013 bar, T1 = 0 + 273 = 273 K p2 = ?, T2 = – 10 + 273 = 263 K Substituting these values in eqn. (i), we get 334 103 (p2 – 1.013 × 10 ) = 1− 1.01 × loge 273 = 334  0 H 263 K . 103 × loge F273I = 12.46 × 105 N/m2 or p2 = 12.46 × 105 + 1.013 × 105 = 13.47 × 105 N/m2 or 13.47 bar. (Ans.) This pressure is considerably high. It can be achieved with ice skate blade by having only a small portion of the blade surface in contact with the ice at any given time. If the temperature drops lower than – 10°C, say – 15°C, then it is not possible to generate sufficient pressure to melt the ice and conventional ice skating will not be possible. Example 7.20. For mercury, the following relation exists between saturation pressure (bar) and saturation temperature (K) : log10 p = 7.0323 – 3276.6/T– 0.652 log10 T Calculate the specific volume v g of saturation mercury vapour at 0.1 bar. Given that the latent heat of vapourisation at 0.1 bar is 294.54 kJ/kg. Neglect the specific volume of saturated mercury liquid. Solution. Latent heat of vapourisation, hfg = 294.54 kJ/kg (at 0.1 bar) Using Clausius-Claperyon equation dp h fg h fg dT v gT (vg −v )T Since vf is neglected, therefore eqn. (i) becomes dp fg dT vgT ...(given) ...(i) Now, log 10 p = 7.0323 – 3276.6 – 0.652 log 10 T dharm M-thermTh7-2.pm 5 = h v . z h = z p f T h F I fg v H T1 K F I 5 b g 263 H K 0 1 = = f f = h T
  • 37. THERMO D Y N A M IC RELATION S 373 Differentiating both sides, we get 1 dp 3276.6 0.652 2.302p dT T 2.302 T or dT = 2.302 × 3276.6 × T2 – 0.652 T ...(ii) From (i) and (ii), we have or fg vgT = 2.302 × 3276.6 × T2 – 0.652 T ...(iii) We know that At p = 0.1 bar, log 10 p = 7.0323 – 3276.6 – 0.652 log 10 T ... (given) log10 (0.1) = 7.0332 – 3276.6 – 0.652 log10 T – 1 = 7.0323 – 3276.6 – 0.652 log10 T or 0.652 log10 T = 8.0323 – 3276.6 or log10 T = 12.319 – 5025.4 Solving by hit and trial method, we get T = 523 K Substituting this value in eqn. (iii), we get 294.54 103 vg 523 = 2.302 × 3276.6 × 0.1105 – 0.652 × 0.1105 (523) 563.17 = 275.75 – 12.46 g i.e., vg = 2.139 m3/kg. (Ans.) HIGHLIGHTS 1. Maxwell relations are given by FTI = – FpJ ; FTJ = F s I v s H  H  G I G I F p I FsI F v I F sI T v v T T p p T 2. The specific heat relation s are cp – cv = vT2 ; cv = T F s I v ; cp = T G T J p . 3. Joule-T homson co-efficien t is expressed as = H p Kh . dharm M-thermTh7-2.pm 5 T T . = – dp p p h p p T T T 2 523 v H  G K J s K p K v s H G K J v p HG KJ  HG KJ H  − H G K G K J J ; . F I K H K G J s T H K GF  T JI
  • 38. 374 ENGINEERIN G THERMOD Y N AM ICS 4. Entropy equation s (Td s equatio n s) : Tds = c dT + T F p I dv v Tds = cpdT – T H T K p dp 5. Equatio n s for interna l energy and enthalpy : F F H H I I v K T K G u J = T G p J – p T v F p I du = c dT + Tv −pV dv SR T G p K = v – T G v J R U dh = cpdT + |v T T p| dp ...(1) ...(2) ...(1) ...[1 (a)] ...(2) ...[2 (a)] | | OBJECTIVE TYPE QUESTIONS Choose the Correct Answer : 1. The specific heat at constan t pressure (c ) is given by (a) cp = T F s I p (b) cp = T F T I p (c) cp = T F T I p (d) cp = T F T I p . 2. The specific heat relation is (a) (cp – cv) = vT2 (c) (cp – cv) = pTK 3. The relation of intern al energy is (a) du = F K cv I F I G J dp + H cp −pK dv (c) du = H  cpJ dp + G v −vJ dv 4. Tds equatio n is (a) Tds = cvdT + T dv (b) (cp – cv) = vTK (d) (cp – cv) = v T . F H K J v  (b) du = G K c I dp + H cp pK dv GF K JI F HG cv I (d) du = H cpK dp + −pKJ v dv.  (b) Tds = cpdT – T dv (c) Tds = cvdT + TK dv (d) Tds = cvdT + T dp. Answers 1. (a) 2. (a) 3. (a) 4. (a). dharm M-thermTh7-2.pm 5 F I   v H  T K U W F H I T F I H K p v − F  H G I K S J V K  2  2 K H  K v HG  KJ T G v J G J  h J T T W p H H G K G K J J T s H H G K G K v J v J G J v  GF K I K c F p I H K 2 v  F I G J  K K  K
  • 39. THERMO D Y N A M IC RELATION S 375 1. Define the co-efficien t of : (i) Volume expansion EXERCISES (ii ) Isothermal compressibil ity (iii ) Adiabatic compressibili ty. 2. Derive the Maxwell relation s and explain their importance in thermo dy namics. 3. Show that the equatio n of state of a substance may be written in the form dv = – Kdp + dT. v 4. A substance has the volume expansivity and isothermal compressibi li ty : = T ; K = p Find the equation of state. NAns. pv constantQ 5. For a perfect gas, show that the difference in specific heats is cp – cv = T . 6. For the followin g given differen tial equatio n s, du = Tds – pdv and dh = Tds + vdp prove that for perfect gas equation , G u J = 0 and T F H G h J = 0. T 7. Using the cyclic equation, prove that HTKv = KT . 8. Prove that the change in entropy is given by ds = cv L KT . dp  M  Q p O dv. 9. Deduce the followin g thermo dy namic relation s : (i) Fh I = v – T G v J = – c FT I (ii ) Fu I = T F p I – p. T p h T v 10. Show that for a Van der Waals gas cp – cv = 1 −2a (v −b)2 / RTv3 . 11. A gas obeys p(v – b) = RT, where b is positive constan t. Find the expression for the Joule-Thomson co-efficien t of this gas. Could this gas be cooled effectiv ely by throttlin g ? 12. The pressure on the block of copper of 1 kg is increased from 10 bar to 1000 bar in a reversible process maintainin g the temperatu re constant at 15°C. Determin e : (i) Work done on the copper durin g the process (ii ) Change in entropy (iii ) The heat transfer (iv) Change in internal energy (v) (cp – cv) for this change of state. The followin g data may be assumed : Volume expansivity () = 5 × 10–5/K Isothermal compressibi lity (K) = 8.6 × 10–12 m2/N Specific volume (v) = 0.114 × 10–3 m3/kg [Ans. (i) – 4.9 J/kg ; (ii ) – 0.57 J/kg K ; (iii ) – 164 J/kg ; (iv) – 159.1 J/kg ; 9.5 J/kg K] dharm M-thermTh7-2.pm 5 F I 1 1 L O M T P R H  p K  I p K GF  p JI  T c N  v P F H I H G J p  K K G H K J G J G J T p p  H  v K H  T K R